Está en la página 1de 112

Universidad de El Salvador

Facultad de Ciencias Naturales y Matemática


Escuela de Matemática
Profesorado en Tercer Ciclo y Educación
Media para la Enseñanza de la Matemática.
GEOMETRÍA I
Ciclo I 2019
Equipo de Diseño:

Eduardo Arnoldo Aguilar Cañas, Oscar Armando Hernández Morales,


Héctor Enmanuel Alberti Arroyo, Francisco Asdrubal Hernández Ramı́rez,
Carlos Mauricio Canjura Linares, Ernesto Américo Hidalgo Castellanos,
Gabriel Alexander Chicas Reyes, Manuel Alejandro Mundo Dueñas,
Claudia Patricia Corcio López de Beltrán, Aarón Ernesto Ramı́rez Flores,
Juan Agustı́n Cuadra, Humberto Alfonso Sermeño Villalta

Resumen
En el presente apunte se examinan los conceptos geométricos básicos, y a partir de estos
se construye el modelo denominado Geometrı́a Euclidiana. Las temáticas se desarrollan de la
forma más elemental y tienen una base importante en la observación y la construcción.
El estudio de la Geometrı́a y especı́ficamente de este modelo es importante, porque en pri-
mer lugar, desarrolla habilidades vitales para cualquier estudiante de matemática y ciencias,
tales como el descubrimiento de patrones, invariantes y el desarrollo de una lógica aguda; en
segundo lugar, porque se rescata lo que históricamente fue el nacimiento de la Matemática
como un sistema lógico riguroso.
Por otro lado, este documento es un apunte de clase de Geometrı́a Plana que se estudian
en la asignatura Geometrı́a I, desde los conceptos fundamentales y las propiedades básicas
de los triángulos, cuadriláteros y circunferencias, hasta el estudio detallado de las rectas y
puntos notables de un triángulo. Además, se estudian tanto problemas métricos como de in-
cidencia: medidas de segmentos, ángulos, áreas, paralelismo, perpendicularidad, concurrencia
y colinealidad. Y, ha sido elaborado con la colaboración de ex alumnos del Programa Jóvenes
Talento y docentes de la escuela de Matemática. Esta recopilación de información extraı́da
de distintos libros, cubre en un buen porcentaje los contenidos exigidos en el programa de
estudio.

1
Geometrı́a I. ÍNDICE

Índice
1. Aspectos históricos 4

2. Segmentos 7
2.1. Introducción . . . . . . . . . . . . . . . . . . . . . . . . . . . . . . . . . . . . . . . . 7
2.2. Lı́neas . . . . . . . . . . . . . . . . . . . . . . . . . . . . . . . . . . . . . . . . . . . 7
2.3. Rayos . . . . . . . . . . . . . . . . . . . . . . . . . . . . . . . . . . . . . . . . . . . 8
2.4. Segmento de recta . . . . . . . . . . . . . . . . . . . . . . . . . . . . . . . . . . . . 8
2.5. Punto medio de un segmento . . . . . . . . . . . . . . . . . . . . . . . . . . . . . . . 8
2.6. Operaciones con segmentos colineales . . . . . . . . . . . . . . . . . . . . . . . . . . 9
2.7. Ejercicios . . . . . . . . . . . . . . . . . . . . . . . . . . . . . . . . . . . . . . . . . 11
2.8. Ejercicios . . . . . . . . . . . . . . . . . . . . . . . . . . . . . . . . . . . . . . . . . 13

3. Ángulos 14
3.1. Ángulos formados por dos rectas y una trasversal a ellas. . . . . . . . . . . . . . . . 16
3.2. Problemas . . . . . . . . . . . . . . . . . . . . . . . . . . . . . . . . . . . . . . . . . 17

4. Triángulos 22
4.1. Clasificación de Triángulos . . . . . . . . . . . . . . . . . . . . . . . . . . . . . . . . 22
4.2. Teoremas Fundamentales del Triángulo . . . . . . . . . . . . . . . . . . . . . . . . . 23
4.3. Perpendicularidad . . . . . . . . . . . . . . . . . . . . . . . . . . . . . . . . . . . . . 24
4.4. Rectas Notables de un triángulo . . . . . . . . . . . . . . . . . . . . . . . . . . . . . 27
4.5. Ejercicios . . . . . . . . . . . . . . . . . . . . . . . . . . . . . . . . . . . . . . . . . 27
4.6. Congruencia de Triángulos. . . . . . . . . . . . . . . . . . . . . . . . . . . . . . . . 33
4.6.1. Criterios de Congruencia de triángulos . . . . . . . . . . . . . . . . . . . . . 33
4.6.2. Teorema de la Base Media . . . . . . . . . . . . . . . . . . . . . . . . . . . . 34
4.6.3. Ejercicios . . . . . . . . . . . . . . . . . . . . . . . . . . . . . . . . . . . . . 38

5. Cuadriláteros 44
5.1. Clasificación de Cuadriláteros (De acuerdo a sus diagonales) . . . . . . . . . . . . . 44
5.2. Paralelogramos . . . . . . . . . . . . . . . . . . . . . . . . . . . . . . . . . . . . . . 45
5.3. Rectángulos . . . . . . . . . . . . . . . . . . . . . . . . . . . . . . . . . . . . . . . . 47
5.4. Rombos . . . . . . . . . . . . . . . . . . . . . . . . . . . . . . . . . . . . . . . . . . 47
5.5. Trapezoides . . . . . . . . . . . . . . . . . . . . . . . . . . . . . . . . . . . . . . . . 49
5.6. Trapecios . . . . . . . . . . . . . . . . . . . . . . . . . . . . . . . . . . . . . . . . . 49
5.7. Problemas . . . . . . . . . . . . . . . . . . . . . . . . . . . . . . . . . . . . . . . . . 51

6. La Circunferencia 54
6.1. Elementos de la circunferencia . . . . . . . . . . . . . . . . . . . . . . . . . . . . . . 54
6.2. Ángulos en la circunferencia . . . . . . . . . . . . . . . . . . . . . . . . . . . . . . . 55
6.3. Cuadriláteros Concı́clicos . . . . . . . . . . . . . . . . . . . . . . . . . . . . . . . . . 56
6.4. Rectas y Circunferencias tangentes a una circunferencia . . . . . . . . . . . . . . . . 58
6.5. Problemas . . . . . . . . . . . . . . . . . . . . . . . . . . . . . . . . . . . . . . . . . 61

UES-FCNM-Escuela de Matemática-Profesorado en Matemática para Tercer Ciclo de Educación


Básica y Educación Media pág. 2
Geometrı́a I. ÍNDICE

7. Semejanza de Triángulos. 71
7.1. Proporcionalidad . . . . . . . . . . . . . . . . . . . . . . . . . . . . . . . . . . . . . 71
7.2. Teorema de Thales . . . . . . . . . . . . . . . . . . . . . . . . . . . . . . . . . . . . 71
7.3. Criterios de Semejanza de Triángulos . . . . . . . . . . . . . . . . . . . . . . . . . . 74
7.4. Potencia de Punto . . . . . . . . . . . . . . . . . . . . . . . . . . . . . . . . . . . . 76
7.5. Problemas . . . . . . . . . . . . . . . . . . . . . . . . . . . . . . . . . . . . . . . . . 78

8. ISOMETRÍAS 87
8.1. Traslaciones. . . . . . . . . . . . . . . . . . . . . . . . . . . . . . . . . . . . . . . . . 87
8.1.1. Propiedades de las Traslaciones. . . . . . . . . . . . . . . . . . . . . . . . . . 87
8.2. Rotaciones. . . . . . . . . . . . . . . . . . . . . . . . . . . . . . . . . . . . . . . . . 90
8.2.1. Rotación en el plano orientado. . . . . . . . . . . . . . . . . . . . . . . . . . 90
8.2.2. Propiedades Esenciales. . . . . . . . . . . . . . . . . . . . . . . . . . . . . . . 91
8.3. Simetrı́as. . . . . . . . . . . . . . . . . . . . . . . . . . . . . . . . . . . . . . . . . . 94
8.3.1. Simetrı́as Centrales . . . . . . . . . . . . . . . . . . . . . . . . . . . . . . . . 94
8.3.2. Propiedades de las simetrı́as centrales. . . . . . . . . . . . . . . . . . . . . . 94
8.3.3. Simetrı́as Ortogonales . . . . . . . . . . . . . . . . . . . . . . . . . . . . . . 96
8.3.4. Propiedades de las simetrı́as ortogonales . . . . . . . . . . . . . . . . . . . . 96
8.4. La Isometrı́a Directa más simple. . . . . . . . . . . . . . . . . . . . . . . . . . . . . 97
8.5. Descomposición de Traslaciones y Rotaciones como Composición de Simetrı́as Axiales. 98
8.6. Problemas de Traslaciones. . . . . . . . . . . . . . . . . . . . . . . . . . . . . . . . . 99
8.7. Problemas de Simetrı́as Centrales o Reflexiones Puntuales. . . . . . . . . . . . . . . 101
8.8. Problemas de Rotaciones. . . . . . . . . . . . . . . . . . . . . . . . . . . . . . . . . 102
8.9. Problemas de Reflexiones Axiales. . . . . . . . . . . . . . . . . . . . . . . . . . . . . 107

9. Rectas y Planos 113


9.1. Posiciones relativas entre rectas y planos. . . . . . . . . . . . . . . . . . . . . . . . . 113
9.2. Rectas coplanares y rectas alabeadas. . . . . . . . . . . . . . . . . . . . . . . . . . . 113
9.3. Recta paralela a un plano. . . . . . . . . . . . . . . . . . . . . . . . . . . . . . . . . 113
9.4. Recta perpendicular a un plano. . . . . . . . . . . . . . . . . . . . . . . . . . . . . . 114
9.5. Planos paralelos, planos secantes y planos perpendiculares. . . . . . . . . . . . . . . 114
9.6. Teorema de las Tres Perpendiculares. . . . . . . . . . . . . . . . . . . . . . . . . . . 114
9.7. Teorema de Thales en el espacio. . . . . . . . . . . . . . . . . . . . . . . . . . . . . 114
9.8. Ángulo entre un plano y una recta secante al plano. . . . . . . . . . . . . . . . . . . 115
9.9. Ángulo diedro. . . . . . . . . . . . . . . . . . . . . . . . . . . . . . . . . . . . . . . 115
9.10. Medida de un ángulo diedro. . . . . . . . . . . . . . . . . . . . . . . . . . . . . . . . 116

UES-FCNM-Escuela de Matemática-Profesorado en Matemática para Tercer Ciclo de Educación


Básica y Educación Media pág. 3
Geometrı́a I. 1 Aspectos históricos

1. Aspectos históricos
Euclides y Arquı́medes son las dos figuras más importantes de la Matemática griega. Mientras
que Arquı́medes es el investigador por excelencia, que incrementa de forma muy considerable el
caudal matemático griego, la trascendente tarea de Euclides estriba en estructurar el patrimonio
matemático griego en un entramado consistente, claramente organizado mediante una concatena-
ción lógica de los resultados – Los Elementos –.
A Euclides le cabe el inmenso mérito de la ordenación y sistematización de la Geometrı́a griega
elemental, de manera que con independencia de sus aportes originales, su mayor contribución se le
reconoce como gran compilador y creador de un estilo de exposición –el método axiomático–, de
modo que en lenguaje actual dirı́amos que Euclides es un gran maestro y su obra fundamental un
Libro de Texto, que establece un férreo paradigma de exposición y de demostración en Matemáticas,
una especie de norma académica de obligado respeto para todo matemático.
Las referencias más fiables sobre Euclides serı́an las que relata Proclo (siglo V d.C.) en su
Comentario al Libro I de Los Elementos de Euclides.
Aproximadamente 300 años antes de Cristo Euclides de Alejandrı́a escribió un tratado en trece
libros llamado Los Elementos. Proclo nos dice: “ Euclides, el autor de los Elementos ordenó diversos
trabajos de Eudoxo, mejoró los de Teeteto y produjo también demostraciones irrefutables para
aquello que sus predecesores no habı́an probado de manera rigurosa. Vivio en la época del primer
Tolomeo, quién una vez le preguntó si habı́a en geometrı́a un camino más corto que los elementos,
y él respondió que no hay un camino real a la geometrı́a.”
Los Elementos de Euclides constan de 465 Proposiciones organizadas en trece libros. De los
trece libros, los primeros seis se pueden describir como los que tratan con triángulos, rectángulos,
cı́rculos, polı́gonos, proporción y semejanza respectivamente. Los Libros VII, VIII y IX tratan de
Teorı́a de Números, es decir de las propiedades de los números enteros y la divisibilidad. El Libro X
introduce el Método de Exhaución y clasifica de forma sistemática los segmentos √ inconmensurables.
En estos últimos dos libros se prueba que hay infinitos números primos, y que 2 es irracional. Los
Libros XI y XII estudian la geometrı́a de sólidos aplicando el Método de Exhaución de Eudoxo al
cálculo del área del cı́rculo y algunos volúmenes. Finalmente el Libro XIII está dedicado al estudio
exhaustivo de cinco poliedros regulares.
Ası́ pues, aunque no hay ninguna introducción o preámbulo de la obra, previamente, en el Libro
I, Euclides introduce unos preliminares a base de veintitrés definiciones, cinco postulados y cinco
nociones comunes o axiomas.

Definición 1.1

1. Punto es lo que tiene posición pero no dimensiones.

2. Lı́nea es la longitud sin anchura.

3. Los extremos de la lı́nea son puntos.

4. Lı́nea recta es la que yace por igual sobre sus puntos.

5. Superficie es lo que sólo tiene largo y ancho.

6. Los extremos de la superficie son lı́neas.

7. Superficie plana es la que yace por igual sobre sus rectas.

UES-FCNM-Escuela de Matemática-Profesorado en Matemática para Tercer Ciclo de Educación


Básica y Educación Media pág. 4
Geometrı́a I. 1 Aspectos históricos

8. Ángulo plano es la inclinación de dos lı́neas que se encuentran en un plano y no yacen las
dos sobre una recta.

9. Si las dos lı́neas que contienen el ángulo son rectas, el ángulo se llama rectilı́neo.

10. Si una recta trazada sobre otra forma con ella dos ángulos contiguos iguales cada uno de
ellos es recto, y la recta se llama perpendicular a aquella sobre la cual se trazó.

11. Ángulo obtuso es el mayor que el recto.

12. Ángulo agudo es el menor que el recto.

13. Lı́mite es el extremo de algo.

14. Figura es lo comprendido por uno o varios lı́mites.

15. Cı́rculo es una figura plana limitada por una sola lı́nea que se llama periferia [circunferencia],
respecto de la cual son iguales las rectas que inciden sobre ellas trazadas desde uno de los
puntos situados en el interior de la figura.

16. Este punto se llama centro del cı́rculo.

17. Diámetro del cı́rculo es una recta cualquiera que pase por el centro y cuyas dos partes tengan
sus extremos en la periferia. Esa recta divide al cı́rculo en dos partes iguales.

18. Semicı́rculo es la figura limitada por un diámetro y la periferia. El centro del semicı́rculo es
el mismo que el del cı́rculo.

19. Figuras rectilı́neas son las limitadas por rectas. Triláteras si lo están por tres, cuadriláteras
por cuatro y multiláteras por más de cuatro.

20. Entre las figuras triláteras el triángulo es equilátero si tiene los tres lados iguales, isósceles
si solo tiene dos lados iguales y escaleno si sus tres lados son desiguales.

21. Entre la figuras triláteras, el triángulo rectángulo es el que tiene un ángulo recto; obtusángulo,
el que tiene un ángulo obtuso, y acutángulo, el que tiene sus tres ángulos agudos.

22. Entre las figuras cuadriláteras, el cuadrado es equilátero y equiángulo; el rectángulo, equiángu-
lo, pero no equilátero; el rombo es equilátero, pero no rectangular; el romboide, sin ser
equilátero ni equiángulo, tiene iguales los lados y los ángulos opuestos. Las demás figuras
cuadriláteras se llaman trapecios.

23. Rectas paralelas son las que, estando en el mismo plano y prolongadas al indefinidamente,
no se encuentran.

Con base en estas definiciones, Euclides presenta a continuación una lista de cinco postulados
y cinco nociones comunes (o axiomas).

UES-FCNM-Escuela de Matemática-Profesorado en Matemática para Tercer Ciclo de Educación


Básica y Educación Media pág. 5
Geometrı́a I. 1 Aspectos históricos

Postulados
1. (Es posible) trazar una lı́nea recta desde un punto cualquiera a otro punto cualquiera.
(Cuando se considera una lı́nea recta contenida entre dos puntos fijos, que son sus extremos,
tal porción se llama una lı́nea recta finita).

2. (Es posible) prolongar de una manera ilimitada en lı́nea recta una recta limitada.

3. (Es posible) describir un cı́rculo para cada centro y cada radio.

4. Todos los ángulos rectos son iguales.

5. Si una recta, al incidir sobre otras dos, forma del mismo lado ángulos internos menores que
dos rectos, las dos rectas prolongadas indefinidamente se encontrarán en el lado en que estén
los ángulos menores que dos rectos.

Nociones Comunes:(Axiomas)
Cosas iguales a una misma cosa son iguales entre sı́.

Si a cosas iguales se agregan cosas iguales, los totales son iguales.

Si de cosas iguales se quitan cosas iguales, los restos son iguales.

Las cosas que se superponen una a la otra son iguales entre sı́.

El todo es mayor que la parte.

La primera de estas Nociones Comunes es la ley transitiva que podemos considerar como el
silogismo fundamental de la Geometrı́a. Las dos siguientes se refieren a la legitimidad de sumar y
restar cosas iguales. La última introduce la desigualdad. La cuarta de las Nociones Comunes merece
una atención. Euclides viene a decir que si una figura se puede trasladar sobre el plano de modo
que al colocarse sobre otra, ambas figuras coinciden perfectamente –se superponen–, entonces las
dos figuras son iguales en todos sus aspectos, es decir, tiene los mismos ángulos, los mismos lados
y demás elementos.

UES-FCNM-Escuela de Matemática-Profesorado en Matemática para Tercer Ciclo de Educación


Básica y Educación Media pág. 6
Geometrı́a I. 2 Segmentos

2. Segmentos
2.1. Introducción
Una figura geométrica es una combinación de puntos, lı́neas y planos. Los conceptos de punto,
lı́nea y plano son conceptos primitivos, que no se definen, únicamente pueden describirse. Por
ejemplo, un punto es un objeto que solo tiene posición. No tiene longitud, anchura ni espesor. Se
trata pués, de una idealización matemática.
La Geometrı́a es la ciencia que estudia la posición, forma y magnitud de las figuras geométricas.
La geometrı́a plana estudia las figuras cuyos puntos se encuentran en el mismo plano (triángulos,
cuadrados, cı́rculos, etc.). La geometrı́a sólida estudia las figuras que pueden estar en distintos
planos (pirámides, cubos, esferas, etc.).

2.2. Lı́neas
Una lı́nea es una figura geométrica que tiene longitud, pero no tiene anchura ni grosor.

Figura 1: Una linea recta

Se puede considerar que una lı́nea se genera con el movimiento de un punto.


Con este enfoque, una lı́nea recta se genera con el movimiento de un punto que se mueve siempre
en la misma dirección (ver Figura 1), mientras que una lı́nea curva se genera con el movimiento
de un punto que cambia continuamente de dirección (ver Figura 2).

Figura 2: Una linea curva

Otro punto de vista consiste en considerar una lı́nea como formada por infinitos puntos. En
particular, una lı́nea recta (llamada por brevedad recta) queda completamente determinada por
dos cualesquiera de sus puntos y se considera ilimitada en extensión.
Notación: Si A y B son dos puntos que pertenecen a una recta, dicha recta se denota como
←→
AB.

UES-FCNM-Escuela de Matemática-Profesorado en Matemática para Tercer Ciclo de Educación


Básica y Educación Media pág. 7
Geometrı́a I. 2 Segmentos

2.3. Rayos
Dada una lı́nea recta, a una porción de dicha recta que se origina en un punto O y que se
extiende ilimitadamente por el extremo que contiene a un punto A, se le llama rayo OA (ver
−→
Figura 3) y se denota por OA.

Figura 3: rayo OA

2.4. Segmento de recta


Dados dos puntos distintos A y B en una recta, se le llama segmento a la figura formada por
A, B y todos los puntos que se encuentran entre ellos dos (ver Figura 4).

A B

Figura 4: Segmento AB

Se denota por AB (o simplemente AB) y se lee “segmento AB”. Los puntos A y B se llaman
extremos y los otros puntos forman el interior del segmento.
La medida de un segmento AB se denota m(AB) o simplemente AB y es un número positivo
que se compara con la longitud de un segmento unitario.

2.5. Punto medio de un segmento


Un punto C se llama punto medio de un segmento AB si C está entre A y B y se verifica que
AC = CB (ver Figura 5).

A C B

Figura 5: C punto medio del segmento AB

Todo segmento posee un punto medio el cual lo biseca, es decir, lo divide en dos segmentos de
igual longitud.

UES-FCNM-Escuela de Matemática-Profesorado en Matemática para Tercer Ciclo de Educación


Básica y Educación Media pág. 8
Geometrı́a I. 2 Segmentos

x
A C B

a b

Figura 6:

2.6. Operaciones con segmentos colineales


Medir es comparar una magnitud con otra de su misma clase que sirve como patrón y a la que
se llama unidad de medida. Para medir los segmentos se utilizan diversos instrumentos, siendo el
más sencillo una regla graduada. En la Figura 6 se cumplen las siguientes relaciones:

AB = AC + CB ⇒ x=a+b (1)

AC = AB − CB ⇒ a=x−b

CB = AB − AC ⇒ b=x−a
Observación:
La relación de adición (1) se puede generalizar ası́: Tomemos “n” puntos consecutivos A1 , A2 , A3 , . . . , An
en una misma recta, entonces se verificará la siguiente relación:

A1 An = A1 A2 + A2 A3 + A3 A4 + · · · + An−1 An

Este resultado se conoce como el Teorema de Chasles. Debemos notar que los segmentos consi-
derados son consecutivos.

Ejemplo 2.1 Ejemplos de problemas con segmentos

a) Sobre una recta están ubicados los puntos A, B, C y D. Si AD = 24 cm, AC = 15 cm y


BD = 17 cm, ¿cuánto mide BC en cm?
Solución
Según los datos del problema (ver Figura 7), se tiene que:
CD = AD − AC = 24 − 15 = 9 cm
y puesto que: BC = BD − CD, se tiene que BC = 17 − 9 = 8 cm. 

b) Sean A, B, C y D puntos consecutivos de una recta. Si AC + BD = 16 m, y BC = 4 m,


¿cuál es el valor de AD en m?
Solución
Construimos la figura según las condiciones del problema y se observa que:
AD = AC + CD, pero CD = BD − 4.
z }| {
Luego, AD = AC + BD −4 = 16 − 4 = 12 m 

UES-FCNM-Escuela de Matemática-Profesorado en Matemática para Tercer Ciclo de Educación


Básica y Educación Media pág. 9
Geometrı́a I. 2 Segmentos

A B C D

Figura 7:

c) M , A, O y B son puntos consecutivos sobre una recta, siendo O el punto medio de AB. Si
M A = 2 y AB = 6, calcular (M O)2 .
Solución
Ya que O es el punto medio de AB, se tiene:
AB 6
AO = OB = = =3
2 2
Luego, M O = M A + AO = 2 + 3 = 5
Por lo tanto, (M O)2 = 25 
d) Sobre una recta se tienen los puntos consecutivos A, B y D. Entre los puntos B y D se toma
un punto C tal que AC = CD/4. Determinar BC, sabiendo que BD − 4AB = 20.
Solución
CD
Sea x = BC. Del dato AC = 4
y llamando AC = a, se tiene CD = 4a.
Reemplazando los literales x y a en la igualdad BD − 4AB = 20, se tiene (dibujar la figura):
(x + 4a) − 4(a − x) = 20
x + 4a − 4a + 4x = 20
5x = 20
x=4
Es decir, BC = 4. 
e) Al dividir un segmento en partes cuyas medidas son directamente proporcionales a 1/3, 1/4
y 1/2 se obtienen tres segmentos, el segundo de los cuales mide 12 cm. ¿Cuál es la suma en
cm de las medidas del segundo y tercer segmento?
Solución
Sean los segmentos AB, BC y AC. Según las condiciones del problema:

k k k
AB = , BC = y CD =
3 4 2
k
ya que BC = 12 entonces 4
= 12, de donde k = 48.
48
Esto nos permite calcular el valor del tercer segmento CD, ası́ CD = 2
= 24.
Luego, la suma pedida es BC + CD = 36. 

UES-FCNM-Escuela de Matemática-Profesorado en Matemática para Tercer Ciclo de Educación


Básica y Educación Media pág. 10
Geometrı́a I. 2 Segmentos

2.7. Ejercicios
1) Sobre una recta se consideran los puntos consecutivos A, B y C. Si AB = 8 cm y BC = 12 cm,
hallar AC.

2) Dados los puntos colineales y consecutivos A, B, C y D, tales que AB = 7, BC = 8 y


AD = 24, calcular CD.

3) En una lı́nea recta se consideran los puntos consecutivos A, B y C tales que AC = 25 y


BC = 15. Calcular AB.

4) A, B, C y D son puntos ubicados en una lı́nea recta de modo que AB = BC, CD = 20 y


AB = 5. Hallar AD.

5) Sobre una linea recta se toman los los puntos consecutivos A, B, C y D. Si M es punto medio
de AD y además AB + CD = 10, BM − M C = 2. Hallar CD

6) Dado el segmento AB y su punto medio O, si P es un punto interior al segmento OB, OP = 1


y P B = 5, calcular AB.

7) En una lı́nea recta se ubican los puntos consecutivos A, B, C y D de modo que AB = 2BC
y CD = 3BC. Si BC = 1, calcular AD.

8) Los puntos A.B y C se encuentran sobre una lı́nea recta, de modo que AB.BC = 41 AC 2 .
AB
Hallar BC + BC
AB

9) Se sabe que los puntos A, B, C, D, E y F son colineales y consecutivos. B es el punto medio


de AF y E lo es de BF ; además 3CD = DF , 3DF = AF y CE = 7/36. Calcular la longitud
de AF .

Definición 2.1 1. Razón: se llama razón, al cociente de dos cantidades, expresadas en la


a
misma magnitud, por ejemplo .
b
a c
2. Proporción: se llama proporción a la igualdad de dos razones. Por ejemplo = , a los
b d
términos a y d se les llama extremos y los términos b y c se les llama medios, al término d
se le llama cuarta proporcional entre a, b y c en este orden.
PA
3. Un punto P ∈ AB divide al segmento AB en una razón r si = r. Si r = 1 entonces P
PB
1
es el punto medio de AB.

A P B

Figura 8:

4. Sean AB y CD y sean X ∈ AB y Y ∈ CD, decimos que X e Y dividen a AB y CD en


segmentos proporcionales si
XA YC
=
XB YD

UES-FCNM-Escuela de Matemática-Profesorado en Matemática para Tercer Ciclo de Educación


Básica y Educación Media pág. 11
Geometrı́a I. 2 Segmentos

A X B

C Y D

Figura 9:

Ejemplo 2.2
A 8 P 2 B

PA 8
P divide AB en la razón = =4
PB 2
Consideremos las divisiones siguientes:
PA x 1
A x P 2x B = =
PB 2x 2

N 3x A 3x B NA 3x 1
= =
NB 6x 2

Dado un segmento AB y una razón k 6= 1, conseguimos encontrar dos puntos que dividen AB
en esta razón: una interior y otra exterior. Cuando AB está dividido por dos punto P y N , en la
misma razón, decimos que el segmento AB está dividido armónicamente. Y, los puntos P y N se
llaman conjugados armónicos con respecto a A y B.
Definición 2.2 ( División armónica)
Decimos que los puntos P y N dividen armónicamente al segmento AB cuando
PA NA N A P B
= .
PB NB
Esta definición de división armónica es equivalente a los sigueinte: Se dice que dos puntos
dividen un segmento de lı́nea armónicamente si lo dividen interna y externamente en la misma
razón.
Definición 2.3 La sección o división áurea es la división armónica de un segmento en media y
extrema razón, es decir, que la parte menor es a la parte mayor, como ésta última es a la totalidad
del segmento.
Tomemos un segmento de longitud uno y hagamos en él la división áurea. Obtenemos una ecua-
ción de segundo grado, cuya solución positiva es el llamado Número de Oro o Número Áureo,
representado por el sı́mbolo griego Φ (Phi).

x 1−x

1
Cuando el puntos P está entre los puntos A y B decimos que P divide interiormente al segmento AB en la
razón r. Si P no está entre los puntos A y B, decimos que P divide exteriormente a segmento AB.

UES-FCNM-Escuela de Matemática-Profesorado en Matemática para Tercer Ciclo de Educación


Básica y Educación Media pág. 12
Geometrı́a I. 2 Segmentos

2.8. Ejercicios
1. Sobre una linea recta se consideran los puntos colineales y consecutivos A, B, C y D; tal que
AC = 19 y BD = 23. Encuentre la longitud del segmento que une los puntos medios de AB
y CD.

2. Sobre una linea recta se consideran los puntos colineales y consecutivos A, B, C y D; siendo
C el puto medio de BD; CB CA
= 32 y AD = 12. Encuentre CD.

3. Sobre una lı́nea recta se consideran los puntos consecutivos A, B, C, D, E y F ; sabiendo que:
AB = EF = BE 3
. Hallar BE, si además: AD + BC + CE + DF = 24.

4. Sobre una lı́nea recta se consideran los puntos consecutivos M, N, P, Q tal que: P Q = 3N P
y 3M N + M Q = 4. Encuentre la longitud del segmento M P

5. Sobre una lı́nea recta se consideran los puntos consecutivos A, B y C y luego se ubican
los puntos medios M √y F de AB y M C respectivamente. Encuentre la longitud de AF . Si
AB + F C − AM = 2 5.

6. Sobre una lı́nea recta se consideran los puntos consecutvos A, B, C, D y luego se toman M y
F puntos medios de AB y CD respectivamente. Encuentre M F , si: AC = 18 y BD = 34.

7. Los puntos A.B y C se encuentran sobre una lı́nea recta, de modo que AB.BC = 14 AC 2 .
AB
Hallar BC + BC
AB

8. Dado el segmento AB y un punto M interior a el. Demuestre que si el producto AM · M B,


es máximo entonces M es el punto medio de AB.

9. Sobre una linea recta se toman los los puntos consecutivos A, B, C y D. Luego se toman los
puntos medios M de AB y N de CD, tal que M N = k(AD + BC). Hallar el valor de k.

10. A, B, C y D son puntos colineales y consecutivos tal que: AB ·CD = AD ·BC; AB ·.BC = x;
AD · CD = y. Calcule BD.

11. Sobre una ı́nea recta se consideran los puntos consecutivos A, B, C, D y D tal que: DC =
2 · AB; AB = a y BD = b. Encuentre AC.

12. A, B, C y D son puntos colineales


 y consecutivos.
 Si AC es la media proporcional2 entre AD
AD AB
y BD. Calcular k, si: k = 2 −1 .
AC CD
13. Se tienen los puntos consecutivos A, A1 , A2 , ..., An sobre una linea recta, tal que AA1 = 1,
A1 A2 = 12 , A2 A3 = 14 , A3 A4 = 81 , ... Calcular AAn

14. Sean A, C, B y D cuatro puntos conjugados armónicos y sea O el punto medio de AB.
Demostrar que OCOD = OA2

15. Dados 4 puntos ordenados A, C, B y D sobre una recta, demostrar que si OCOD = OA2 con
O el punto medio de AB entonces A, C, B y D forman una cuarteta armónica.

2
Dados dos segmentos de longitudes a y b se llama media proporcional a un segmento de longitud x, talque
a x
verifique = .
x b

UES-FCNM-Escuela de Matemática-Profesorado en Matemática para Tercer Ciclo de Educación


Básica y Educación Media pág. 13
Geometrı́a I. 3 Ángulos

3. Ángulos
Definición 3.1 Definimos como ángulo a la figura geométrica formada por dos rayos (o semirrec-
tas) distintas que tienen el mismo origen. Ese origen se llama vértice del ángulo. Al ángulo de
vértice O y rayos OA y OB se le denota ∠AOB.

B
B
B

A
O

A A
A
O B
O O

Ángulo agudo Ángulo recto


Ángulo obtuso

Figura 10: Ejemplo de ángulos

Dos ángulos ∠AOB y ∠BOC son adyacentes si y sólo si tienen un lado común OB y los lados
no comunes OA y OC están en semiplanos distintos, determinados por el lado común.

A
O

Figura 11: Ejemplo de angulos adyacentes

Bisectriz de un ángulo es la semirrecta que lo “divide” en dos ángulos adyacentes iguales.

La construcción con regla y compás de la bisectriz, se hace de la siguiente manera:


Hacemos centro en el vértice O del ángulo y abriendo el compás la medida que se quiera, se traza
un arco que corta a los lados del ángulo en los puntos C y D, Ahora tomamos una abertura del
compás, un poco mayor a la mitad del arco comprendido entre los puntos C y D. La abertura puede
ser cualquiera pero tiene que ser un poco mayor a la mitad del citado arco. Con esta abertura se
trazar un arco desde C y otro desde D. Se cortan en el punto E. Uniendo E con el vértice O del
ángulo, obtenemos la BISECTRIZ

UES-FCNM-Escuela de Matemática-Profesorado en Matemática para Tercer Ciclo de Educación


Básica y Educación Media pág. 14
Geometrı́a I. 3 Ángulos

Figura 12:

Dos ángulos son:

Congruentes o Iguales: si tienen igual medida.

Suplementarios: si su suma es 180°.

Complementarios: si su suma es 90°.

Por otra parte, dos rectas en el plano pueden ser secantes o paralelas,3 dependiendo si se cortan
o no; además, si las rectas son secantes, el punto de corte es único, y definen cuatro ángulos, que
se agrupan por parejas en ángulos opuestos por el vértice (las parejas de ángulos tales que uno
está formado por la prolongación de los lados del otro).

C
B

D A

Los ángulos opuestos por el vértice son iguales (Justifique). Por ejemplo, ∠AOB = ∠COD.
Por lo que dos rectas secantes forman cuatro ángulos que definen dos parejas de ángulos iguales,
y si tomamos un miembro de cada pareja, se tienen dos ángulos suplementarios. En particular, si
las rectas son secantes y forman cuatro ángulos iguales, serán llamadas rectas perpendiculares,4 y
los ángulos ası́ generados son llamados ángulos rectos. Y como es muy conocido, un ángulo agudo
es aquel cuya medida es menor a la de un ángulo recto, y un ángulo obtuso es aquel cuya medida
es mayor que un ángulo recto; en particular, un ángulo obtuso será llamado ángulo llano si su
medida es el doble que la de un ángulo recto.

3
Si la recta AB es paralela a la recta CD, se denota AB k CD.
4
Si la recta AB es perpendicular a la recta CD, se denota AB ⊥ CD.

UES-FCNM-Escuela de Matemática-Profesorado en Matemática para Tercer Ciclo de Educación


Básica y Educación Media pág. 15
Geometrı́a I. 3 Ángulos

3.1. Ángulos formados por dos rectas y una trasversal a ellas.


Al intersecar un par de rectas por una recta llamada transversal o secante, se forman los
siguientes tipos de ángulo:

β
α
γ θ
L1

ε φ
L2
ψ δ

Ángulos internos: γ, θ, ε, φ

Ángulos externos: α, β, ψ, δ

Ángulos Correspondientes: Son dos ángulos no adyacentes situados en el mismo lado de la


secante, uno en el interior y otro en el exterior: α y ε; β y φ; θ y δ; γ y ψ.

Ángulos Alternos Internos: Son dos ángulos no adyacentes situados en el interior, y en dis-
tintos lado de la secante: γ y φ; θ y ε.

Ángulos Alternos Externos: Son dos ángulos no adyacentes situados en el exterior, y en


distintos lado de la secante: α y δ; β y ψ.

Ángulos Conjugados: Son dos ángulos internos o externos, no adyacentes y situados del
mismo lado de la secante:

a) Ángulos conjugados internos: γ y ε; θ y φ.


b) Ángulos conjugados externos: α y ψ; β y δ.

Cuando las rectas L1 y L2 son paralelas (L1 k L2 ) se cumple que:

UES-FCNM-Escuela de Matemática-Profesorado en Matemática para Tercer Ciclo de Educación


Básica y Educación Media pág. 16
Geometrı́a I. 3 Ángulos

1. Los ángulos correspondientes son igua- L


les entre sı́.
β
2. Los ángulos alternos internos son igua- α
les entre sı́. L1 γ θ

3. Los ángulos alternos externos son igua-


les entre sı́. ε φ
L2
4. Los ángulos conjugados son suplemen- ψ δ
tarios.

3.2. Problemas
1. Tres ángulos adyacentes forman un semiplano y tienen sus medidas proporcionales a los
números 5, 7 y 8. Hallar la medida del menor ángulo.

2. Demostrar que las bisectrices de dos ángulos suplementarios son perpendiculares.

3. En la figura adjunta, L1 k L2 y
L3 k L4 . Calcular x.

−−→ −−→ −→
4. Se tienen los ángulos consecutivos AOB, BOC y COD, siendo OM , ON y OL las bisectrices
de los ángulos AOB, COD y M ON respectivamente. Hallar la medida del ángulo COL, si
m]M OC − m]N OD = 80◦ .

5. Con ayuda de la figura 13, demuestre que: Si L1 k L2 entonces γ = α + β.

6. En la figura 14, AB k F G. Hallar el ángulo x si el ∠AM F = 90° y el ∠M AB = 110°.

7. Calcular el ∠OP Q, si OP es bisectriz del ángulo O, L1 k L2 y P Q ⊥ L1 . Ver figura 15.

UES-FCNM-Escuela de Matemática-Profesorado en Matemática para Tercer Ciclo de Educación


Básica y Educación Media pág. 17
Geometrı́a I. 3 Ángulos

Figura 13:

Figura 14:

Figura 15:

8. En la figura 16, L1 k L2 y L3 k L4 , calcular α.

UES-FCNM-Escuela de Matemática-Profesorado en Matemática para Tercer Ciclo de Educación


Básica y Educación Media pág. 18
Geometrı́a I. 3 Ángulos

Figura 16:

9. En la figura 17, X, O y Y son colineales y OP, OS, OQ son las bisectrices de ∠XOA, ∠AOB
←→ ←→
y ∠BOY . Calcular el ángulo ∠P OS si OS ⊥ XY y ∠XOA + ∠QOY = 120◦

Figura 17:

10. En la figura 18, calcular x, si L1 k L2 .

Figura 18:

UES-FCNM-Escuela de Matemática-Profesorado en Matemática para Tercer Ciclo de Educación


Básica y Educación Media pág. 19
Geometrı́a I. 3 Ángulos

11. Calcular la medida θ del gráfico anexo, si las


rectas L1 y L2 son paralelas.

12. En la figura ??, ∠BOD = 80◦ y ∠AOD − ∠AOB = 12◦ . Halle la medida del ∠BOC

Figura 19:

13. En la figura 20, L1 k L2 y L3 k L4 . Hallar el valor del ángulo θ.

Figura 20:

14. Sea ∠AOB = 24°, en la región exterior a dicho ángulo se traza el rayo OC. Hallar la medida
del ángulo formado por las bisectrices de los ángulos AOC y BOC.

15. En la figura L1 k L2 , si α + β + γ + θ = 122◦ hallar x

16. Del gráfico 21, calcular y, cuando x tome su máximo valor entero.

UES-FCNM-Escuela de Matemática-Profesorado en Matemática para Tercer Ciclo de Educación


Básica y Educación Media pág. 20
Geometrı́a I. 3 Ángulos

x+y

2y − x x−y

Figura 21:

17. Si L1 k L2 , calcular α + β + γ.

UES-FCNM-Escuela de Matemática-Profesorado en Matemática para Tercer Ciclo de Educación


Básica y Educación Media pág. 21
Geometrı́a I. 4 Triángulos

4. Triángulos
Diremos que tres puntos que pertenecen a una misma recta son puntos colineales; de manera
análoga, si tres rectas pasan por un mismo punto, serán llamadas rectas concurrentes. Si tomamos
“al azar” tres puntos en el plano, en muy raras ocasiones estos puntos estarán alineados,5 y diremos
entonces que son los vértices de un triángulo; análogamente sucede con las rectas, tres rectas por
lo general no concurren, y la figura geométrica que éstas definen es también un triángulo.6 Una
definición completa para nuestros intereses es la siguiente:

Definición 4.1 (Definición de Triángulo)

Si A, B y C son tres puntos cualesquiera no colineales (Ver figura 54), entonces la reunión se los
segmentos AB, BC y AC se llama triángulo ABC y se denota por 4ABC. Los puntos A, B y
C se llaman vértices y los segmentos AB, BC y AC se llaman lados. Simbólicamente: 4ABC =
AB ∪ BC ∪ AC.

4.1. Clasificación de Triángulos


1. Con relación a sus lados:

a) Escaleno: si sus tres lados no son congruentes.


b) Isósceles: si por lo menos dos de sus lados son congruentes.
c) Equilátero: si sus tres lados son congruentes (note un triángulo equilátero es también
isósceles, y que los tres ángulos internos son iguales entre sı́ e iguales a 60)

2. Con relación a sus ángulos internos:

a) Acutángulo: si su ángulo mayor es agudo (note que entonces los tres ángulos son agudos)
b) Rectángulo: si su ángulo mayor es ángulo recto (note que el ángulo en cuestión es único
y que los otros dos ángulos son agudos; ası́, en un triángulo rectángulo, la hipotenusa
es mayor a los catetos)
c) Obtusángulo, si el ángulo mayor es ángulo obtuso (note que el ángulo en cuestión es
único y que los otros son agudos; ası́, en un triángulo obtusángulo, el lado que se opone
al ángulo obtuso es el lado mayor)

Todo triángulo ABC determina tres ángulos internos o interiores: ∠ABC, ∠ACB y ∠BAC,
y se llamará ángulo externo o exterior, al ángulo determinado por un lado y la prolongación del
lado adyacente, en la figura 54, α, β y θ son ángulos exteriores.
Dado el 4ABC, se tiene que AB + BC + CA = p = 2s, donde p es llamado el perı́metro y s el
semiperı́metro del triángulo. Para abreviar, suele asociarse a cada vértice un lado opuesto, y vice-
versa, por ejemplo, el lado opuesto de A es BC, y es frecuente que se denote por a; análogamente
b = CA, c = AB.

5
En teorı́a de probabilidades, ¡la probabilidad que esto ocurra es cero!
6
El término más riguroso para esta figura es trilátero. En este caso, habrı́a que hacer una consideración: si hay
un par de rectas paralelas, el trilátero definido ya no es “normal” según nuestro sentido común, sin embargo, ¡sigue
siendo un trilátero!

UES-FCNM-Escuela de Matemática-Profesorado en Matemática para Tercer Ciclo de Educación


Básica y Educación Media pág. 22
Geometrı́a I. 4 Triángulos

Figura 22: Elementos del Triángulo

4.2. Teoremas Fundamentales del Triángulo


(Cada teorema lo demostraremos en clase)

Teorema 1 En todo triángulo, la medida de un ángulo exterior es igual a la suma de las medidas
de dos ángulos interiores del triángulo no adyacentes a él.

La demostración de este teorema se basa en las relaciones de ángulos entre paralelas; se deja al lec-
tor que haga la demostración (Sugerencia: por un vértice, trace una recta paralela al lado opuesto)

Teorema 2 En todo triángulo, la suma de las medidas de sus tres ángulos internos es igual a
180°.

Teorema 3 (Desigualdad Triangular) En todo triángulo, la longitud de uno de sus lados está com-
prendido entre la suma y la diferencia de los otros dos.

Sin ser muy rigurosos, suponga que dado el segmento AB se traza con centro en A una circun-
ferencia de radio r1 , y con centro en B una circunferencia de radio r2 ; si AB < r1 + r2 , las circunfe-
rencias se cortarán en dos puntos, y cualquiera de ellos puede ser el vértice C, ası́ AB < BC + CA;
en cambio, si AB = r1 + r2 o peor aún, si AB > r1 + r2 , la construcción del 4ABC no es posible.

La Desigualdad Triangular es un resultado fundamental, a partir de ésta y de su modelo de de-


mostración se generan los Criterios de Congruencia de Triángulos; a groso modo, si dadas ciertas
condiciones, la construcción de una figura geométrica (un triángulo en particular) queda deter-
minada de manera única, entonces dos figuras que reunen las mismas condiciones serán llamadas
figuras congruentes.

Ası́, si se tienen tres segmentos (cuyas longitudes cumplen la desigualdad triangular), dejando
uno fijo y construyendo las circunferencias con centros en los extremos de este segmento y radios
las longitudes de los otros segmentos, por construcción, sólo será posible obtener dos triángulos
(uno con cada punto de intersección de las circunferencias), que son básicamente el mismo pero
la orientación de los ángulos es contraria; ası́, si se sabe que dos triángulos cumplen tener lados
respectivamente iguales, por construcción, deben de ser iguales. Este es el conocido criterio LLL

UES-FCNM-Escuela de Matemática-Profesorado en Matemática para Tercer Ciclo de Educación


Básica y Educación Media pág. 23
Geometrı́a I. 4 Triángulos

de congruencia de triángulos; más adelante se detallarán el resto de criterios, pero a partir de este
probaremos el siguiente resultado:

Teorema 4 En todo triángulo, se cumple que a lados iguales se oponen ángulos iguales, y vice-
versa.

Suponga que 4ABC es tal que AB = AC, entonces, por criterio LLL, 4ABC es congruente
al 4ACB (en ese orden, porque AB = AC, BC = CB y CA = BA), entonces, los ángulos que se
oponen a los ángulos iguales son iguales. Para el recı́proco necesitamos otro criterio de congruen-
cia, por lo que la demostración se dejará incompleta; retome esto en la sección de congruencia de
triángulos.

Teorema 5 En todo triángulo se cumple que a mayor lado se opone mayor ángulo y viceversa.

Este teorema se deja como ejercicio para el lector (Sugerencia: utilice el teorema anterior, tome
el lado mayor y defina un punto adecuado que genere un triángulo con dos lados iguales.)

4.3. Perpendicularidad
Mediatriz de un segmento
Se llama mediatriz del segmento AB a la recta que es perpendicular a este segmento y que pasa
por su punto medio. La mediatriz divide al segmento AB en otros dos segmentos de igual longitud.
La recta mediatriz tiene una importante propiedad: la distancia de cualquier punto de esa recta a
cada uno de los dos extremos del segmento AB es la misma.
Construcción de la mediatriz.
Vamos a construir la mediatriz de un segmento utilizando, como en casos anteriores, la regla y
el compás. Para ello representa dos puntos y traza el segmento que los une utilizando la regla.
Coloca el compás sobre uno de los extremos del segmento y ábrelo para que coincida con el otro
extremo. Traza ası́ una circunferencia. Haz la misma operación apoyando el compás sobre el otro
extremo. Une ahora los puntos donde se cortan las dos circunferencias que acabas de trazar. El nue-
vo segmento es perpendicular al inicial y si lo prolongas obtendrás la recta mediatriz que buscabas.

Figura 23:

UES-FCNM-Escuela de Matemática-Profesorado en Matemática para Tercer Ciclo de Educación


Básica y Educación Media pág. 24
Geometrı́a I. 4 Triángulos

Distancia de un punto a una recta. En la figura 24, sea P un punto exterior a una recta
L, la longitud de la perpendicular P M a la recta L es la distancia del punto P a dicha recta. Esta
perpendicular tiene la propiedad de ser única y su longitud es la distancia mı́nima del punto a la
recta (Pruébelo utilizano el hecho que la hipotenusa es mayor que los catetos). Los segmentos P A
y P B no son perpendiculares a L y se llaman oblicuas.

Figura 24:

Altura de un triángulo.
la altura es la menor distancia entre un vértice y el lado opuesto (o su prolongación), por lo que a
cada vértice le corresponde una altura. También utilizamos el nombre de altura para referirnos a
la recta que pasa por un vértice y es perpendicular al lado opuesto, pues es sobre esta recta sobre
la que medimos esa distancia.
Construcción de la altura.
Con C como centro y un radio suficientemente grande, construya un arco que corte a AB en P y Q.
Con P y Q como centros y un radio mayor que la mitad de P Q, construya arcos que se intersecten
en R. Trace CR que intersecta a AB en N. CN es la altura con respecto al lado AB.

Figura 25:

Teorema de Pitágoras
Abordamos el estudio de las Relaciones Métricas, del cual solo realizaremos el análisis del famoso
Teorema de Pitágoras, cuyo enunciado es el siguiente:

UES-FCNM-Escuela de Matemática-Profesorado en Matemática para Tercer Ciclo de Educación


Básica y Educación Media pág. 25
Geometrı́a I. 4 Triángulos

Teorema 6 (Teorema de Pitágoras) En un triángulo rectángulo, el cuadrado de la hipotenusa es


igual a la suma de los cuadrados de los catetos.

Una demostración de este teorema es debida a Thabit ibn Qurra (836-901), la cual consiste en
diseccionar la figura que se forma al construir dos cuadrados de lados respectivamente iguales a los
catetos de un triángulo rectángulo, como se muestra en el gráfico 26. A continuación se presentan

Figura 26:

otras pruebas del Teorema de Pitágoras.

Figura 27:

Figura 28:

UES-FCNM-Escuela de Matemática-Profesorado en Matemática para Tercer Ciclo de Educación


Básica y Educación Media pág. 26
Geometrı́a I. 4 Triángulos

Teorema 7 (Recı́proco del teorema de Pitágoras) Si en un triángulo el cuadrado de un lado es


igual a la suma de los cuadrados de los otros dos lados, el triángulo es rectángulo.7 .

4.4. Rectas Notables de un triángulo


1. Altura: Se llama altura de un triángulo al segmento que parte de uno de sus vértices y llega
en forma perpendicular al lado opuesto o a su prolongación.

2. Mediana: Se llama Mediana al segmento que une un vértice con el punto medio del lado
opuesto.

3. Mediatriz: Se denomina mediatriz de un lado de un triángulo es la recta perpendicular a


dicho lado en su punto medio.

4. Una Bisectriz: La bisectriz es la recta que “divide” en dos ángulos iguales a un ángulo dado;
en particular, es bisectriz interna si es la bisectriz de un ángulo interno de un triángulo, y
bisectriz externa si es la bisectriz de un ángulo externo de un triángulo.

Figura 29:

4.5. Ejercicios

1. En la figura adjunta ambos triángulos son


equiláteros. Encuentre el valor de ϕ.

7
Ver demostración en la sección de congruencia de triángulos (página 34)

UES-FCNM-Escuela de Matemática-Profesorado en Matemática para Tercer Ciclo de Educación


Básica y Educación Media pág. 27
Geometrı́a I. 4 Triángulos

2. En la figura 30, calcular el ∠x si el ∠AOB = 100° y L1 k L2 .

Figura 30:

3. (*) En la figura 31, ABDE es un cuadrado y BCD es un triángulo isósceles con BD = DC.
Si ∠ABC = 160°, determinar la medida de ∠AEC.

Figura 31:

4. (*) (XV Competencia de Clubes Cabri Primera Ronda) En la figura


adjunta, ABCD es un rectángulo tal que AB = 2BC. M es el punto
medio de AB y los triángulos AM E y M BF son equiláteros. Si P
es la intersección de las rectas DE y CF , encuentre los ángulos del
4CDP .

5. Probar que una bisectriz exterior de un triángulo es paralela al lado opuesto si y sólo si el
triángulo es isósceles.
θ
6. Si AB y F G son rectas paralelas, el ∠ABC = ∠CDE = θ, el ∠DEF = 2
y el ∠GF H = 150°.
Calcule θ. Figura 32

UES-FCNM-Escuela de Matemática-Profesorado en Matemática para Tercer Ciclo de Educación


Básica y Educación Media pág. 28
Geometrı́a I. 4 Triángulos

Figura 32:

7. (*) Hallar la suma de los ángulos α +  + θ + φ en la figura 33.

Figura 33:

8. Determine el valor de la suma ∠A + ∠B + ∠I + ∠H + ∠F + ∠G. Figura 34.

Figura 34:

9. En el 4ABC el ∠BAC = 36° y AC = AB. Probar que la bisectriz interior BD (D en AC)


es congruente con el lado BC.

UES-FCNM-Escuela de Matemática-Profesorado en Matemática para Tercer Ciclo de Educación


Básica y Educación Media pág. 29
Geometrı́a I. 4 Triángulos

10. Sea ABC un triángulo rectángulo en B con AB = BC, se construye exteriormente el triángu-
lo equilátero BCD. Encuentre el ángulo ∠DAB.

11. En el 4ABC, AB = AC y D un punto sobre la recta AC, tal que BC = BD = DA.


Determine la medida del ángulo ∠ABD, si:

a) D está entre A y C.
b) A está entre D y C.

12. En un 4ABC, D es un punto sobre el lado AC tal que AB = AD. Si ∠ABC −∠ACB = 90°,
hallar el ∠CBD.

13. Se tiene un triángulo isósceles ABC, AB = BC en el cual se traza al altura AF tal que
BF = 6 y F C = 2. Hallar AC.

14. En la figura 35, el ∠ABC = ∠ACE, DC = EC, ¿Qué lı́nea notable es AD del 4BCA?

Figura 35:

15. ¿Cuál es el valor de b − a en la figura 36?

Figura 36:

16. (*) Sea ABC un triángulo tal que las medianas respectivas a B y C son perpendiculares.
Demuestre que se cumple la relación.

5BC 2 = CA2 + AB 2 .

UES-FCNM-Escuela de Matemática-Profesorado en Matemática para Tercer Ciclo de Educación


Básica y Educación Media pág. 30
Geometrı́a I. 4 Triángulos

17. La hipotenusa BC de un triángulo rectángulo ABC se divide en 4 segmentos congruentes


por los puntos G, E y H. Si BC = 20, encuentra la suma de los cuadrados de las longitudes
de los segmentos AG, AE y AH. Figura 37.

Figura 37:

18. (*) Dado un cuadrado ABCD, se construyen los triángulos equiláteros ABP (exteriormente)
y ADQ (interiormente). Probar que C, P y Q están alineados.

19. (*) Sea ABC un triángulo rectángulo con ∠CAB = 90°. D es un punto sobre la prolongación
de BC tal que BD = BA. E es un punto en el mismo semiplano que A respecto de BC, tal
que CE ⊥ BC y además CE = CA. Mostrar que A, D y E están alineados.

20. El cuadrilátero ABCD mostrado en la figura 38 cumple que AB k CD y BC k DA.8 Sobre las
prolongaciones de AB y AD se construyen puntos E y F tales que BC = BE y DC = DF .
Demuestre que C, E y F están alinedos.

Figura 38:

21. (*) En la figura adjunta, AB = BC = CD =


DE = EF = F G = GA. Calcule la medida del
∠DAE.

8
El cuadrilátero ABCD es un paralelogramo.

UES-FCNM-Escuela de Matemática-Profesorado en Matemática para Tercer Ciclo de Educación


Básica y Educación Media pág. 31
Geometrı́a I. 4 Triángulos

22. (*) (XXVIII Olimpiada Brasileña de Matemática) En la figura 39, AB = AC, AM = AN y


∠CAM = 30°, encuentre el valor del ∠BM N .

Figura 39:

23. Los lados de un triángulo isósceles son 12 y 5 metros, ¿cuál es su perı́metro?


a+b+c
24. Muestre que los lados de un triángulo cumplen que |a − b| < c y que c < 2
.

25. Muestre que es posible construir un triángulo con segmentos de longitudes a, b, c si y sólo
existen números positivos x, y, z tales que: a = x + y, b = y + z, c = z + x.

26. (*) (Etapa semifinal Estatal de XXII Olimpiada Mexicana de Matemáticas) En la figura
40 se muestra un hexágono regular ABCDEF de lado 1. Los arcos del cı́rculo que están
dibujados tienen centro en cada vértice del hexágono y radio igual a la distancia al vértice
opuesto. P , Q, R, S, T y U son los puntos de corte de estos arcos. ¿Cuánto mide cada lado
del hexágono P QRST U ?

Figura 40:

UES-FCNM-Escuela de Matemática-Profesorado en Matemática para Tercer Ciclo de Educación


Básica y Educación Media pág. 32
Geometrı́a I. 4 Triángulos

4.6. Congruencia de Triángulos.


Definición 4.2

El 4ABC es congruente al 4A0 B 0 C 0 si: AB = A0 B 0 , AC = A0 C 0 , BC = B 0 C 0 , ∠ABC = ∠A0 B 0 C 0 ,


∠ACB = ∠A0 C 0 B 0 y ∠BAC = ∠B 0 A0 C 0 . Simbólicamente: 4ABC = 4A0 B 0 C 0 . Véase figura 41.

Figura 41: Definición de Igualdad de Triángulos.

La definición anterior establece que dos triángulos son congruentes si tanto los lados como los
ángulos se presentan en pares respectivos congruentes. Esto, según la visión de Euclides, significa
que un triángulo es posible superponerlo sobre el otro (se puede desplazar, girar o reflejar) y coin-
cidirá de manera perfecta. Sin embargo, es importante mencionar que en muy raras ocasiones se
tendrá a disposición tanta información, de allı́ la importancia de los criterios de congruencia, que
establecen los requisitos mı́nimos para garantizar que dos triángulos son congruentes.

4.6.1. Criterios de Congruencia de triángulos


El siguiente es el primero de los tres criterios de congruencia de triángulos, y se denomina
criterio de LADO-ÁNGULO-LADO, en sı́mbolos: L-A-L.

Criterio L-A-L. Si los triángulos ABC y A0 B 0 C 0 presentan las congruencias: AB = A0 B 0 ,


AC = A0 C 0 y ∠BAC = ∠B 0 A0 C 0 , entonces 4ABC = 4A0 B 0 C 0 .

Figura 42: Criterio LAL

Según el criterio L-A-L, dos triángulos son congruentes si en uno de ellos existen dos lados y
el ángulo (comprendido entre dichos lados), respectivamente congruentes a dos lados y el ángulo
(comprendido entre dichos lados), en el otro triángulo.

Criterio A-L-A. Sean ABC y A0 B 0 C 0 dos triángulos tales que: AC = A0 C 0 , ∠BCA = ∠B 0 C 0 A0


y ∠BAC = ∠B 0 A0 C 0 , entonces 4ABC = 4A0 B 0 C 0 .

UES-FCNM-Escuela de Matemática-Profesorado en Matemática para Tercer Ciclo de Educación


Básica y Educación Media pág. 33
Geometrı́a I. 4 Triángulos

Figura 43: Criterio ALA.

Criterio L-L-L. Si un triángulo tiene sus tres lados respectivamente congruentes a los tres
lados de otro triángulo, entonces estos dos triángulos son congruentes.

Figura 44: Criterio LLL.

Ahora demostraremos el Recı́proco del Teorema de Pitágoras(página 27).

Demostración: Sea ABC un triángulo talque BC 2 = AB 2 + AC 2 , por construcción sea el


4A0 B 0 C 0 rectángulo en A0 tal que A0 B 0 = AB y A0 C 0 = AC, entonces por el teorema de Pitágoras
B 0 C 02 = A0 B 02 + A0 C 02 , ası́ que B 0 C 02 = BC 2 , de donde B 0 C 0 = BC y por el criterio LLL, se deduce
que el 4A0 B 0 C 0 = 4ABC, por lo tanto el ∠BAC = ∠B 0 A0 C 0 = 90°.

4.6.2. Teorema de la Base Media


En todo triángulo, el segmento que une los puntos medios de dos lados es paralelo al tercer
lado e igual a su mitad.

Figura 45: Teorema de La Base Media.

En la figura 45, M N es el segmento que une los puntos medios de los lados AB y BC del 4ABC,
AC
a este segmento se le llama BASE MEDIA DEL TRIÁNGULO. Se verifica que M N = y que
2

UES-FCNM-Escuela de Matemática-Profesorado en Matemática para Tercer Ciclo de Educación


Básica y Educación Media pág. 34
Geometrı́a I. 4 Triángulos

M N k AC.
Demostración:

1. Prolongar el segmento M N hasta el punto P tal que M N = N P .

2. Los triángulos M N B y P N C son congruentes, ya que BN = N C, M N = N P y el ∠CN P =


∠M N B, por consiguiente, el ∠N CP = ∠M BN , por lo tanto, CP k M B (Por ángulos
alternos internos iguales). Además, P C = M B = M A; con lo cual se tiene que: M A = P C.

3. Uniendo el punto A con el punto P se forman los triángulos congruentes AM P y ACP (por
L A L) ya que M A = P C, AP = AP , ∠M AP = ∠AP C (por ángulos alternos internos
entre las paralelas M A y P C). Luego, M P = AC, entonces N P = 12 M P = 12 AC. Además,
∠P AC = ∠M P A, de donde M P k AC o que M N k AC.

Corolario: Menor mediana de un triángulo rectángulo. En todo triángulo rectángulo,


la mediana relativa a la hipotenusa es la mitad de la longitud de la hipotenusa y es la menor de
las tres medianas del triángulo.

Demostración: En la figura 46, BM es la mediana relativa a la hipotenusa AC del 4ABC,


probaremos que BM = AC 2
; (con lo cual se tendrá que BM = AM = M C). Si por M se traza
una paralela al lado AB, que corte al lado BC en N , entonces N es el punto medio de BC y
el ∠M N C = 90°, los triángulos BN M y CN M son congruentes por el criterio L-A-L, luego
M B = M C = AM .
Para probar que BM es la menor mediana hacemos uso de lo que hemos demostrado anteriormente,
es decir que BM = AM = M C y del hecho que M N es base media y además trazamos la mediana
AN , formándose el triángulo 4AM N .
El 4M CN es rectángulo en N eso implica que los otros dos ángulos son agudos, en particular lo
es ∠CM N en consecuencia el ∠AM N es obtuso, lo cual implica que el lado AN es el mayor lado
en el 4AM N es decir AN < AN = BM . De manera análoga se demuestra para la otra mediana.

Figura 46: Menor Media en un Triángulo Rectángulo.

Ejemplo 4.1 En un triángulo ABC, AB = AC, D está sobre AB y E está sobre la prolongación
de AC tal que BD = CE. Si el segmento DE intersecta a BC en G. Demuestre que DG = GE.

Consideremos la siguiente figura:

UES-FCNM-Escuela de Matemática-Profesorado en Matemática para Tercer Ciclo de Educación


Básica y Educación Media pág. 35
Geometrı́a I. 4 Triángulos

Figura 47: Ejemplo 5.

Por el punto D, se traza la recta DF paralela a AE, con F sobre BC. Luego

∠F DG = ∠CEG, ∠DGF = ∠EGC


y puesto que
∠BF D = ∠BCA = ∠DBF
Ası́
DF = DB = CE
Luego 4DF G ∼
= 4ECG (ALA) entonces

DG = GE.

Ejemplo 4.2 Dado un triángulo ABC equilátero de lado 1, se ubica un punto exterior al lado BC
tal que BD = DC y el ángulo BDC = 120◦ . Se ubican los puntos M y N sobre los lados AB
y AC respectivamente, tal que ∠M DN = 60◦ . Encuentre el perı́metro del 4AM N . Sugerencia:
prolongue AB, hasta un punto P , de forma que BP = N C.

Consideremos la siguiente figura:

UES-FCNM-Escuela de Matemática-Profesorado en Matemática para Tercer Ciclo de Educación


Básica y Educación Media pág. 36
Geometrı́a I. 4 Triángulos

Al prolongar la recta AB hasta el punto P de manera que BP = N C, nos damos cuenta de que
al unir el punto D con el punto P se tiene que los triángulos 4BDP y N CD son congruentes
por el criterio LAL, ya que al ser el ángulo ABC equilátero, cada uno de sus ángulos mide 60◦ y
en el 4BDC, ∠DBC = ∠DCB = 30◦ , ya que el ∠BDC = 120◦ , ası́ ∠P BD = 90◦ = ∠N CD.
Luego, los triángulos M P D y M DN son congruentes, por criterio LAL, ya que P D = DN , el lado
M D es común en ambos triángulos y ∠BDM = 120◦ − 60◦ − α = 60◦ − α. Por tanto el ángulo
∠P DM = α + (60◦ − α) = 60◦ . Por tanto M P = M N . Por tanto se tiene

Perı́metro: 4M AN = M A + M N + N A
= (1 − M B) + M N + (1 − N C)
= 1 − M B + M B + BP + 1 − P B
= 2.

UES-FCNM-Escuela de Matemática-Profesorado en Matemática para Tercer Ciclo de Educación


Básica y Educación Media pág. 37
Geometrı́a I. 4 Triángulos

4.6.3. Ejercicios
1. Sea el 4ABC un triángulo cualquiera. Sobre los lados AB y AC se construyen triángulos
equiláteros ABC 0 y CAB 0 . Pruebe que BB 0 = CC 0 .

2. (*) En la figura adjunta, ABC es un triángulo equilátero


y CDEF es un cuadrado. Se construye un punto G tal
que CF = CG y además ∠CF G = 15°. Probar que
∠AGC = ∠BDC.

3. Dado un triángulo equilátero ABC, se construye un triángulo equilátero DEF cuyos vértices
están sobre los lados del 4ABC, tal como muestra la figura 48. Demuestre que los triángulos
ADF , BED, CF E son todos congruentes entre si.

Figura 48:

4. ABCD es un cuadrado, E, F , G y H son puntos sobre los lados AB, BC, CD, DA, respec-
tivamente, tal que EF GH también es cuadrado. Demuestre que los triángulos AEH, BF E,
CGF , DHG son todos congruentes entre si. Figura 49.

Figura 49:

5. ABCDE y F GHIJ son pentágonos regulares (Vease figura 50). Demuestre que los triángulos
AF J, BGF , CHG, DIH, EJI son todos congruentes entre si.

UES-FCNM-Escuela de Matemática-Profesorado en Matemática para Tercer Ciclo de Educación


Básica y Educación Media pág. 38
Geometrı́a I. 4 Triángulos

Figura 50:

6. Si AB k CD y AB = CD entonces, AD = BC y AD k BC 9 .

7. Demuestre que dos triángulos desplazados


son congruentes. Sugencia: Utilice el pro-
blema anterior.

8. Demuestre que dos triángulos rotados son congruen-


tes.

10
9. Demuestre que dos triángulos reflejados con respecto a un punto son congruentes.

9
El cuadrilátero ABCD se denomina paralelogramo.
10
La reflexión con respecto a un punto es equivalente a una rotación de 180°

UES-FCNM-Escuela de Matemática-Profesorado en Matemática para Tercer Ciclo de Educación


Básica y Educación Media pág. 39
Geometrı́a I. 4 Triángulos

10. Demuestre que dos triángulos reflejados con respecto a


una recta son congruentes.

Importante: Las traslaciones, rotaciones y reflexiones no cambian el tamaño ni la forma de


un triángulo.

11. (*) En la figura adjunta, ABCD un cuadrado y EF ⊥ GH.


Demuestre que que EF = GH.

12. Dos cuadrados ABCD y EHGF , ambos de lado l,


están colocados en manera tal que un vértice de uno
está en el centro del otro (como en la figura anexa).
l2
Demuestre que el área del cuadrilátero EJBK es
4
y por ende no depende de la posición de J (o K).

13. En un 4ABC el ∠B = 2∠C, la mediatriz del lado AC corta en F al lado BC. Hallar AB,
si F C = 9.

14. En la figura 51, AC = 12 AF = 4 y ∠BAF = 30. Hallar BF si AG = GC.

UES-FCNM-Escuela de Matemática-Profesorado en Matemática para Tercer Ciclo de Educación


Básica y Educación Media pág. 40
Geometrı́a I. 4 Triángulos

Figura 51:

15. En la figura 52, AG = GC, el ∠AF G = 20°. Hallar el ∠F AC, si AC = 2BF .

Figura 52:

16. En la figura 4ADB, 4AF C y 4BEC son


triángulos equiláteros; calcular ∠DF E si el ∠ABC
es recto en B.

17. (*) (Examen final de XVI Olimpiada Mexicana de Matemática) Los ángulos de un triángulo
ABC están en progresión aritmética (∠B − ∠A = ∠C − ∠B = θ), D, E, y F son los puntos
medios de los lados BC, CA y AB, respectivamente. Llamamos H al pie de la altura trazada
desde C (que cae entre B y F ) y G a la intersección entre DH y EF . Hallar los ángulos del
4F GH.

UES-FCNM-Escuela de Matemática-Profesorado en Matemática para Tercer Ciclo de Educación


Básica y Educación Media pág. 41
Geometrı́a I. 4 Triángulos

18. (*) Sea ABCD un cuadrado. Se construyen


triángulos equiláteros ADP y ABQ como se mues-
tra en la figura ??. Sea M la intersección de CQ
con AD y N la intersección de CP con AB. De-
muestre que CM N es un triángulo equilátero.

19. En la figura 4ADB, 4AF C y 4BEC son


triángulos equiláteros; calcular ∠DF E si el ∠ABC
es recto en B.

20. Considere la figura adjunta.


I
i) Pruebe que 4ABE es congruente a 4ACF .
G

ii) Pruebe que 4BCD es congruente a 4ACI. B

F
iii) Argumente porqué (AJKE) = (AF GB) y
A C
(CDKJ) = (BHIC). J

iv) Concluya el Teorema de Pitágoras.

Nota: (AJKE) significa el área de la figura


AJKE. De igual manera es para los otros casos.
E D
K

21. En la figura 53, ABC, CDE y EF A son triángulos isósceles, con el ∠ABC = ∠CDE =
∠EF A = 120°. Probar que el 4BDF es equilátero.

UES-FCNM-Escuela de Matemática-Profesorado en Matemática para Tercer Ciclo de Educación


Básica y Educación Media pág. 42
Geometrı́a I. 4 Triángulos

Figura 53:

22. (*) 4ABC es un triángulo isósceles con ∠ABC = ∠ACB = 80°. D es un punto en AC tal
que ∠ABD = 10°. Demuestre que AD = BC.

UES-FCNM-Escuela de Matemática-Profesorado en Matemática para Tercer Ciclo de Educación


Básica y Educación Media pág. 43
Geometrı́a I. 5 Cuadriláteros

5. Cuadriláteros
5.1. Clasificación de Cuadriláteros (De acuerdo a sus diagonales)
Los cuadriláteros pueden clasificarse de acuerdo a sus diagonales de la siguiente forma:

Cuadrilátero Convexo: Es un cuadrilátero con las dos diagonales en su interior.

B
C

Cuadrilátero Entrante: Es un cuadrilátero con una diagonal en el interior y otra en el exterior.

B
C

Cuadrilátero Cruzado Es un cuadrilátero con las diagonales en su exterior.


B
A

11

Es muy frecuente que se considere que un cuadrilátero es convexo, a menos que se especifique
lo contrario. Esto es ası́ porque muchos resultados son más claros en un cuadrilátero convexo, sin
embargo, es importante darse cuenta que existen teoremas que no se cumplen para cualquier tipo
de cuadriláteros, por ejemplo:

Teorema 8 La suma de los ángulos internos de un cuadrilátero no cruzado es 360◦ .

11
Tanto los cuadriláteros convexos como los entrantes son cuadriláteros simples, que son los cuadriláteros cuyos
lados no se cortan salvo en los extrenos; en contraposición, los cuadriláteros cruzados no son simples.

UES-FCNM-Escuela de Matemática-Profesorado en Matemática para Tercer Ciclo de Educación


Básica y Educación Media pág. 44
Geometrı́a I. 5 Cuadriláteros

La demostración de este resultado se basa en la disección del cuadrilátero en dos triángulos


cuyos ángulos internos conforman los ángulos internos del cuadrilátero, sin embargo, estas condi-
ciones no pueden lograrse en un cuadrilátero cruzado; de hecho, la suma de los ángulos internos
puede hacerse arbitrariamente pequeña cuando el cuadrilátero es cruzado.

También hay otras clasificaciones de cuadriláteros de acuerdo a sus lados y ángulos.

Cuadrilátero Equiángulo: un cuadrilátero (convexo) es equiángulo si todos sus ángulos in-


ternos son iguales; dado el teorema anterior, los ángulos son iguales a 90◦ , por ello este cuadrilátero
es llamado rectángulo.

Cuadrilátero Equilátero: un cuadrilátero (convexo) es equilátero si todos sus lados son igua-
les. A este cuadirátero también se le conoce como rombo.

Cuadrado: es un cuadrilátero que es equiángulo y equilátero.

C B

D A

Paralelogramo: es un cuadrilátero con los lados opuestos paralelos.

Trapecio: es un cuadrilátero con un par de lados opuestos paralelos.12

5.2. Paralelogramos
Es el cuadrilátero que tiene sus lados opuestos paralelos y congruentes. En todo paralelogramo
se cumple que sus ángulos opuestos son congruentes y sus diagonales se bisecan. El paralelogramo
también se conoce como romboide.
D
C

A
F B

Dado el paralogramo ABCD, por propiedades de ángulos entre paralelas es posible probar el
siguiente resultado:

12
Note que un paralelogramo es también un trapecio.

UES-FCNM-Escuela de Matemática-Profesorado en Matemática para Tercer Ciclo de Educación


Básica y Educación Media pág. 45
Geometrı́a I. 5 Cuadriláteros

Teorema 9 En todo paralelogramo, los ángulos opuestos son iguales y los ángulos consecutivos
son suplementarios: ∠ABC = ∠CDA = θ y ∠BCD = ∠DAB = 180 − θ.

Por otra parte, por criterio ALA, 4ABC ≡ 4CDA; esto implica que AB = CD y BC = DA,
i.e.

Teorema 10 Los lados opuestos de un paralogramos son iguales.

A partir de esto, si M es la intersección de AC con BD, por criterio ALA, 4ABM ≡ 4CDM ,
por lo que AM = CM y BM = DM , i.e.

Teorema 11 Las diagonales de un paralelogramo se bisecan.

Además, se cumple un resultado sofisticado y muy importante:

Teorema 12 Ley del Paralelogramo. Si ABCD es un paralelogramo entonces el doble de la


suma de los cuadrados de los lados es igual a la suma de los cuadrados de las diagonales, es decir

2 AB 2 + BC 2 = AC 2 + BD2


Demostración: Sea el paralelogramo ABCD, desde los vértices C y D se trazan las distancias
←→
hacia la recta AB, estos segmentos caen en los puntos H 0 y H respectivamente. (ver fig.)

Figura 54: Elementos del Triángulo

Se forman entonces los triángulos rectángulos AH 0 D y BHC los cuales son rectángulos y con-
gruentes ya que DH 0 = CH por ser ambas distancias entre las paralelas, AD = BC por ser un
paralelogramo y además ∠H 0 AD = ∠HBC por lo que ∠H 0 DA = ∠HCB son complementarios de
ángulos congruentes.
Luego usando el teorema de Pitágoras en los triángulos rectángulos AH 0 D, AHC, H 0 BD y BHC

UES-FCNM-Escuela de Matemática-Profesorado en Matemática para Tercer Ciclo de Educación


Básica y Educación Media pág. 46
Geometrı́a I. 5 Cuadriláteros

tenenmos:

AC 2 = AH 2 + HC 2
AC 2 = (AB + BH)2 + HC 2 ...(1)
BD2 = (DH 0 )2 + H 0 B 2
BD2 = (DH 0 )2 + (AB − AH 0 )2 ...(2)

Sumando (1) y (2), tenemos:

AC 2 + BD2 = (AB + BH)2 + HC 2 + (DH 0 )2 + (AB − AH 0 )2


AC 2 + BD2 = (AB + BH)2 + HC 2 + (HC)2 + (AB − BH)2 ya que DH 0 = HC y AH 0 = BH
AC 2 + BD2 = AB 2 + 2AB.BH + BH 2 + HC 2 + HC 2 + AB 2 − 2AB.BH + BH 2
AC 2 + BD2 = 2AB 2 + 2BH 2 + ( (((( + 2HC 2 − (
2AB.BH 2AB.BH
((((
AC 2 + BD2 = 2(AB 2 + BH 2 + HC 2 )
AC 2 + BD2 = 2(AB 2 + BC 2 ) ya que BC 2 = BH 2 + HC 2 en el triángulo rectángulo BHC 

5.3. Rectángulos
Rectángulo: Sus cuatro ángulos son igual a 90°, sus lados opuestos son iguales y paralelos.

D C

A B

En primer lugar, es importante notar que todo rectángulo es paralelogramo (por ángulos entre
paralelas), por lo que todos los resultados probados anteriormente son heredados a todo rectángulo;
pero los rectángulos tienen propiedades adicionales:

Observe que por criterio LAL, 4ABC ≡ 4ABD, por lo que AC = BD y entonces

Teorema 13 Las diagonales de un paralelogramo son iguales; además, el punto de intersección de


estas equidista de los cuatro vértices y por tanto es el centro de una circunferencia que pasa por
todos los vértices.

Por otra parte, observe que si se aplica la ley del paralelogramo a un rectángulo se obtiene el
Teorema de Pitágoras.

5.4. Rombos
Rombo: Sus cuatro lados son iguales.

UES-FCNM-Escuela de Matemática-Profesorado en Matemática para Tercer Ciclo de Educación


Básica y Educación Media pág. 47
Geometrı́a I. 5 Cuadriláteros

A
C

Dado un rombo ABCD, por criterio LLL, 4ABC ≡ 4CDA, y por lo tanto ∠BAC = ∠DAC
y ∠BCA = ∠DAC, lo cual implica BC k AD y AB k CD, i.e., todo rombo ABCD es un
paralelogramo. Además, por las mismas congruencias se tiene

Teorema 14 Las diagonales de un rombo cumplen ser una mediatriz de la otra.

Teorema 15 Las diagonales de un rombo bisecan a los ángulos interiores del rombo; esto implica
que el punto de corte de las diagonales equidista de los cuatro lados del rombo y es el centro de
una circunferencia tangente a estos.

Observaciones.

1. Un cuadrilátero es un para lelogramo si se cumple cualquiera de las siguientes condiciones:

a) Sus lados opuestos son paralelos.


b) Sus lados opuestos son congruentes.
c) Dos lados son congruentes y paralelos.
d ) Sus ángulos opuestos son congruentes.
e) Sus diagonales se bisecan entre sı́.

2. Se cumple la siguiente relación de inclusı́on.

Paralelogramo

Rectángulos Cuadrados
Rombos

UES-FCNM-Escuela de Matemática-Profesorado en Matemática para Tercer Ciclo de Educación


Básica y Educación Media pág. 48
Geometrı́a I. 5 Cuadriláteros

5.5. Trapezoides
Es un cuadrilátero que no tiene pares de lados paralelos. Los trapezoides se clasifican en:
Trapezoide asimétrico: No tiene ningún par de lados paralelos o congruentes.

B
C

Trapezoide simétrico: Dos pares de lados consecutivos son congruentes; además una de las
diagonales es mediatriz de la otra.
B

A C

5.6. Trapecios
Es el cuadrilátero que tiene dos lados paralelos denominados base y los otros dos no son para-
lelos. La distancia entres sus bases se llama altura, y el segmento que une los puntos medios de los
lados no paralelos se denomina mediana.
Sea M y N los puntos medios de AD y BC. AB k DC.

D Base MenorC

E F

Base Mayor B
A G

Los trapecios se clasifican en: Trapecio escaleno: Es aquel en que sus lados no paralelos son
diferentes.

UES-FCNM-Escuela de Matemática-Profesorado en Matemática para Tercer Ciclo de Educación


Básica y Educación Media pág. 49
Geometrı́a I. 5 Cuadriláteros

D C

B
A

Trapecio isósceles13 :Es aquel en sus lados no paralelos son congruentes. AB = CD


D C

B
A

Trapecio rectángulo: Cuando uno de sus lados no paralelos es perpendicular a las bases.

D C

B
A

Dado el trapecio ABCD (con AB k CD), se construyen los puntos medios de BC y DA, M
y N , respectivamente. Si el cuadrilátero M N AB se rota con centro en M y ángulo 180 se genera
un cuadrilátero M N 0 A0 C; observe que N D = N 0 A0 y N D k N 0 A0 , por lo que DN N 0 A0 es un
paralelogramo y
N N 0 = DA0
2M N = DC + CA0
2M N = DC + AB
AB + CD
⇒ MN =
2
El segmento M N es llamado base media del trapecio, y por lo recién demostrado se tiene

Teorema 16 La base media de un trapecio es igual a la semisuma de las bases.

Área de un cuadrilátero: ¿ Qué es el área de una figura plana ?


Vamos a llamar área a la medida de la superficie que hay dentro de una figura en dos dimensiones,
es decir, una figura plana. Por ejemplo, observemos las siguientes figuras:
13
Los trapecios isósceles son muy importantes cuando se estudian los ángulos en la circunferencia; resulta que un
trapecio es isósceles si y sólo si los cuatro vértices se ubican sobre una misma circunferencia.

UES-FCNM-Escuela de Matemática-Profesorado en Matemática para Tercer Ciclo de Educación


Básica y Educación Media pág. 50
Geometrı́a I. 5 Cuadriláteros

Figura 55:

Para calcular el área de una superficie debemos compararla con otra que elegimos como unidad
de superficie, y averiguar el número de unidades que contiene. Teniendo en cuenta la definición que
hemos visto para el área de una figura, podemos aplicarla a figuras sencillas y obtener expresiones
generales para cada una de ellas. Consideremos el siguiente rectángulo de 8cm de ancho y 4cm
de altura. Realizamos un análisis similar al del cuadrado. Si tomamos como unidad de medida la
superficie que ocupa un cuadrado que tiene lados de longitud 1, podemos observar que el cuadrado
pequeño cabe 32 veces en el rectángulo. Entonces el área del rectángulo es igual a 32.

5.7. Problemas
1. Dado el trapecio ABCD con AB k CD, demuestre que la bisectriz interior del ∠A es paralela
a la bisectriz exterior del ∠D.

2. A un rombo ABCD se le construyen exteriormente los cuadrados ABEF y BCGH. De-


muestre que 4ABD = 4EBH.

3. (*) Sea ABCD un paralelogramo. Se construyen triángulos equiláteros exteriores 4CDP y


4ADQ. Demuestre que el 4BP Q es equilátero.

4. Demuestre que las bisectrices interiores de un paralelogramo forman un rectángulo (¿qué su-
cede si el paralelogramo es además rombo?).

5. Demuestre que las bisectrices exteriores de un paralelogramo forman un rectángulo.

6. Sea ABCD un paralelogramo. La bisectriz interna del ∠CDA corta a BA en M , y la bisectriz


interna del ∠BAD corta a CD en N . Demuestre que ADN M es un rombo.

7. Demuestre que si por el punto de intersección de las diagonales de un rombo se trazan perpen-
diculares a los lados del rombo, entonces los puntos de intersección de dichas perpendiculares
con los lados del rombo forman un rectángulo.

8. Demuestre que las bisectrices de los ángulos definidos por las diagonales de un rombo, cortan
a los lados del rombo en cuatro puntos que forman un cuadrado.

9. En un 4ABC sea G la intersección de las medianas BB 0 y CC 0 . Sean B 00 , C 00 las reflexiones


de G respectivas a los puntos B 0 y C 0 .

a) Demuestre que AGCB 00 y AGBC 00 son paralelogramos.


b) A partir de lo anterior, demuestre que BCB 00 C 00 también es paralelogramo.

UES-FCNM-Escuela de Matemática-Profesorado en Matemática para Tercer Ciclo de Educación


Básica y Educación Media pág. 51
Geometrı́a I. 5 Cuadriláteros

c) Demuestre que A0 pertenece a la recta AG, y concluya que las tres medianas de un
triángulo concurren en el punto G, llamado el centroide del 4ABC.
d) Demuestre que CG = 2GC 0 ; relaciones similares se cumplen para las otras dos medianas.
10. Teorema de Varignon: Dado un cuadrilátero ABCD (no necesariamente convexo), se
construyen los puntos medios L, M , N , O, P , Q, de los segmentos de recta AB, BC, CD,
DA, BD, AC, respectivamente. Figura 56.
a) Demuestre que LM N O, LP N Q, OP M Q, son paralelogramos.
b) Demuestre que LN , OM , P Q concurren en un punto, llamado el centroide del cuadrilátero
ABCD.
c) Demuestre que el perı́metro de LM N O es igual a AC + BD; resultados similares se
cumplen para los otros paralelogramos.

Figura 56: Teorema de Varignon

11. Sea ABCD un paralelogramo tal que existe un punto E sobre el lado AB que cumple
∠CED = 90. Sean M y N los pies de las perpendiculares trazadas desde A y B hacia DE
y CE, respectivamente. Demuestre que AC, BD y M N concurren.
12. (*) (Héctor Alberti) Sea ABCD un cuadrado. Se construyen los triángulos equiláteros BDA0 ,
ACB 0 , BDC 0 y ACD0 . Demuestre que el A0 B 0 C 0 D0 es también un cuadrado.
13. (*) (II Olimpiada Matemática del Cono Sur) En la figura 57 ABCD y AECF son paralelo-
gramos. Demuestre que BEDF es paralelogramo.
14. (*) ABCD es un cuadrilátero convexo y O es un punto en su interior. Sean P , Q, R, S, los
puntos medios de los lados AB, BC, CD, DA, respectivamente. Por P se traza una paralela
a OR, por Q se traza una paralela a OS, por R se traza una paralela a OP , y por S se traza
una paralela a OQ. Demuestre que estas cuatro rectas concurren.
15. (*) Un trapecio isósceles tiene diagonales perpendiculares y su área es 2010, determine su
altura.
16. (*) (IX Competencia de Clubes Cabri, Segunda Ronda) Sea ABCDEF un hexágono regular
cuyo centro es O. Se construyen los cuadrados F SOP y ORCQ. Demuestre que AP QB y
SEDR son rectángulos. Figura 58.

UES-FCNM-Escuela de Matemática-Profesorado en Matemática para Tercer Ciclo de Educación


Básica y Educación Media pág. 52
Geometrı́a I. 5 Cuadriláteros

Figura 57:

Figura 58:

17. (*) Sobre los lados del 4ABC se trazan exteriormente los cuadrados ABP Q, CARS y
BCT U . Luego se trazan los paralelogramos AQA0 R, CSC 0 T y BU B 0 P .

a) Sean A00 , B 00 , C 00 los centros de los cuadrados BCT U , CARS, ABP Q, respectivamente.
Demuestre que estos centros están sobre los lados del 4A0 B 0 C 0 .
b) Demuestre que AA00 , BB 00 , CC 00 concurren.

18. (*) Se dibujan cuadrados exteriores a los lados de un paralelogramo, demuestre que:

a) El cuadrilátero determinado por los centros de esos cuadrados es un cuadrado.


b) Las diagonales de ese cuadrado son concurrentes con las del paralelogramo.

19. (*) Dado un 4ABC, se construyen exteriormente los triángulos rectángulo isósceles 4ACP
y 4BCQ, con AC y BC como hipotenusas. Si M es el punto medio de AB, demuestre que
el 4M P Q también es un triángulo rectángulo isósceles.

UES-FCNM-Escuela de Matemática-Profesorado en Matemática para Tercer Ciclo de Educación


Básica y Educación Media pág. 53
Geometrı́a I. 6 La Circunferencia

6. La Circunferencia
6.1. Elementos de la circunferencia
Una circunferencia es el lugar geométrico de puntos que equidistan de un punto dado, llama-
do el centro de la circunferencia; la distancia de cada punto de la circunferencia al centro es el radio.

Por otra parte, todos los puntos que están a una distancia del centro menor o igual al radio
forman el cı́rculo; estos puntos quedan “al interior” o sobre la circunferencia.

Si A y B son dos puntos de una circunferencia, el segmento de recta AB define una cuerda;
en particular, si el centro de la circunferencia pertenece a la cuerda, ésta es llamada diámetro.
Es importante mencionar que para cada punto de la circunferencia existe exactamente un punto
diametralmente opuesto.

Figura 59: O: Centro, OQ: radio, M N : cuerda, AB: diámetro, MN: arco, P Q: flecha.

En la figura 60, se tiene una circunferencia de centro O y radio r = OA = OB = OA0 ; AB y


AA0 son cuerdas, pero AA0 es también diámetro, i.e, A0 es diametralmente opuesto a A y viceversa.
Observe que por la desigualdad triangular aplicada al triángulo isósceles 4AOB

Figura 60:

AB < AO + BO
= r+r
= AA0

UES-FCNM-Escuela de Matemática-Profesorado en Matemática para Tercer Ciclo de Educación


Básica y Educación Media pág. 54
Geometrı́a I. 6 La Circunferencia

Si A es un punto fijo, esta desigualdad es válida para cualquier punto B sobre la circunferencia
(excepto cuando B = A0 lo cual implica AB = AA0 ). Esto quiere decir que el diámetro es la mayor
de todas las cuerdas.

6.2. Ángulos en la circunferencia


A las porciones de circunferencia que quedan entre dos puntos ubicados en la circunferencia,
se les llama arcos de circunferencia; note que dos puntos sobre una circunferencia definen dos
arcos de circunferencia. También, si un ángulo tiene vértice sobre el centro de la circunferencia y
está formado por dos radios, será llamado ángulo central ; de nuevo, ∠AOB hace referencia a dos
ángulos, cuya suma es 360, y subtienden respectivamente a uno de los arcos AB. Finalmente, si un
ángulo tiene el vértice sobre la circunferencia y está formado por dos cuerdas, será llamado ángulo
inscrito; en la figura anterior, ∠AA0 B es un ángulo inscrito que subtiende al arco AB.

Teorema: El ángulo central es el doble del ángulo inscrito que subtiende el mismo arco.

Demostración: Considere la figura 61, se demostrará que ∠AOB = 2∠AP B en los tres casos
mostrados. En la circunferencia de la izquierda, sea P 0 el punto diametralmente opuesto a P ;
observe que 4AP O y 4BP O son triángulos isósceles, y por el teorema del ángulo externo se tiene

∠AOB = ∠AOP 0 + ∠BOP 0


= (∠AP O + ∠OAP ) + (∠BP O + ∠OBP )
= 2∠AP O + 2∠BP O
= 2 (∠AP O + ∠BP O)
= 2∠AP B

Figura 61:

El caso de la circunferencia del medio es más sencillo y se deja como ejercicio para el lector.
Para la circunferencia de la derecha, el trabajo es análogo y sólo cambia en un pequeño arreglo
algebraico

∠AOB = ∠BOP 0 − ∠AOP 0


= (∠BP O + ∠OBP ) − (∠AP O + ∠OAP )
= 2∠BP O − 2∠AP O
= 2 (∠BP O − ∠AP O)
= 2∠AP B

UES-FCNM-Escuela de Matemática-Profesorado en Matemática para Tercer Ciclo de Educación


Básica y Educación Media pág. 55
Geometrı́a I. 6 La Circunferencia

Corolario: Todos los ángulos inscritos que subtienden el mismo arco son iguales (Ver figura
62). En particular, los ángulos internos son iguales a 90° si subtienden a una semicircunferencia.

Figura 62:

Demostración: Todos los ángulos mostrados en la figura 62 son iguales a la mitad del ∠AOB,
y por tanto, son iguales entre sı́. En particular, si AB fuera un diámetro, ∠AOB = 180° y por
tanto ∠AP B = 90°. 14

Hay un par de ángulos más que son importantes: Si un punto P es interno a la circunferencia,
el ángulo de vértice P formado por dos cuerdas que pasan por P se llama ángulo interior. De
forma similar, si P es exterior y dos cuerdas de la circunferencia (al prolongarse) pasan por P , el
ángulo con vértice P es llamado ángulo exterior.

Dejamos como ejercicio demostrar el siguiente teorema:

Teorema: Los ángulos interior y exterior mostrados en la figura 63 cumplen las fórmulas
siguientes:
∠BOD + ∠AOC
∠AQC =
2
∠BOD − ∠AOC
∠AP C =
2

6.3. Cuadriláteros Concı́clicos


Ahora suponga que sobre una circunferencia se ubican cuatro puntos A, B, C, D, como se
muestra en la figura 64. Al cuadrilátero ABCD se le llama cuadrilátero cı́clico o concı́clico. Observe
que
α β
∠ABC + ∠CDA = + = 180◦ .
2 2
Y análogamente ∠DAB +∠BCD = 180°. Esto significa que si ABCD es un cuadrilátero cı́clico
y convexo, entonces los ángulos opuestos son suplementarios. También, es posible demostrar por
14
Observe que en cualquier triángulo rectángulo, el punto medio de la hipotenusa equidista de los tres vértices.

UES-FCNM-Escuela de Matemática-Profesorado en Matemática para Tercer Ciclo de Educación


Básica y Educación Media pág. 56
Geometrı́a I. 6 La Circunferencia

Figura 63:

Figura 64:

contradicción el recı́proco de este resultado: si suponemos que ABCD es tal que ∠B + ∠D = 180
pero no es cı́clico, se define el punto D0 como la otra intersección de AD con el circuncı́rculo del
4ABC, y como ABCD0 es cı́clico (por construcción) entonces ∠B +∠D0 = 180, luego, ∠D = ∠D0 ,
lo cual implica la contradicción CD k CD0 (rectas paralelas que se cortan en C). Ası́, se ha de-
mostrado el siguiente teorema:

Teorema: El cuadrilátero convexo ABCD es un cuadrilátero cı́clico si y sólo si

∠A + ∠C = 180◦ = ∠B + ∠D

También, otro criterio muy útil y cuya demostración también se basa en el corolario anterior
es

Teorema: El cuadrilátero convexo ABCD es un cuadrilátero cı́clico si y sólo si se cumple


alguna de las siguientes igualdades

∠ABD = ∠ACD
∠BCA = ∠BDA
∠BAC = ∠BDC
∠CAD = ∠CBD

Es importante recalcar que NO todo cuadrilátero puede ser inscrito en una circunferencia; por
ejemplo, un paralelogramo no será cı́clico a menos que sea rectángulo.

UES-FCNM-Escuela de Matemática-Profesorado en Matemática para Tercer Ciclo de Educación


Básica y Educación Media pág. 57
Geometrı́a I. 6 La Circunferencia

Teorema de Miquel: Si D, E, F son tres puntos cualesquiera en los lados BC, CA, AB del
triángulo ABC, entonces las circunferencias que pasan por las tercias de puntos B, D, F ; C, E, D; A, F, E;
tienen un punto en común. Ver figura 65

Figura 65:

Prueba: (Ejercicio, Use concı́clicos )


Teorema: La Recta de Simson-Wallace. Sean X, Y y Z los pies de las alturas trazadas
desde un punto P en el circuncı́rculo del 4ABC hacia AB, BC y CA, respectivamente, son
colineales.

6.4. Rectas y Circunferencias tangentes a una circunferencia


Dada una circunferencia, una recta puede ser tangente o secante a la circunferencia, depen-
diendo si la corta en uno o dos puntos, respectivamente; en cualquier otro caso, se dice que la
recta no corta a la circunferencia.15

Sea l una recta secante a la circunferencia que corta a la circunferencia en A y B (A 6= B);


como el 4AOB es isósceles, ∠OAB < 90. Recı́procamente, si por A se traza una recta l tal que
uno de los ángulos que forma con OA es menor que 90, se puede construir un punto B sobre l tal
que ∠OAB = ∠ABO < 90 y A 6= B (basta proyectar O sobre l y luego reflejar A con respecto a
este punto, el resultante es el punto B); entonces el 4AOB es isósceles, por lo que OA = r = OB,
i.e. B pertenece a la circunferencia y por tanto l corta a la circunferencia en dos puntos distintos. Ası́

Teorema: Una recta l corta a una circunferencia de centro O en dos puntos distintos A y B
si y sólo si un ángulo entre l y OA es agudo.

Corolario: Si l es una recta tangente en A a una circunferencia de centro O, ninguno de los


ángulos entre l y OA puede ser agudo, y por tanto l ⊥ OA.

A partir de este resultado se prueban otros resultados muy conocidos y útiles, que dejamos de
ejercicios para el lector.

Teorema: Dado un punto P externo a una circunferencia de centro O, si P A y P B son seg-


mentos tangentes a la circunferencia en A y B, respectivamente, entonces el cuadrilátero P AOB
15
Cuando la recta es tangente a la circunferencia puede considerarse como un caso muy peculiar en el cual los
“dos” puntos de corte coinciden.

UES-FCNM-Escuela de Matemática-Profesorado en Matemática para Tercer Ciclo de Educación


Básica y Educación Media pág. 58
Geometrı́a I. 6 La Circunferencia

es cı́clico y bisósceles.

Corolario: Dado un punto P externo a una circunferencia de centro O, la circunferencia de


diámetro P O corta a la circunferencia dada en dos puntos A y B tales que P A y P B son rectas
tangentes.

Definición: El ángulo semi-inscrito en una circunferencia es aquel que se forma con una cuer-
da y la recta tangente en alguno de los extremos de la cuerda.

Teorema: La media del ángulo semi-inscrito definido por la cuerda AB es igual a la medida
de un ángulo inscrito que subtiende al arco AB.

Demostración: Considere la figura 66. Como AP BO es cı́clico, entonces ∠P AB = ∠P OB;


además, como P O es la mediatiz de AB, ∠P OB = ∠P OA, por lo que
∠AOB
∠P AB = = ∠AQB
2

Figura 66:

Por otra parte, dada una circunferencia, otra circunferencia puede ser secante o tangente a la
primera, dependiendo si la corta en uno o dos puntos, respectivamente; en cualquier otro caso se
dice que las circunferencias no se cortan.16

Además, dos circunferencias pueden posicionarse una dentro de la otra, y claramente, la cir-
cunferencia de radio mayor es la externa mientras que otra es la interna; particularmente, si las
circunferencias tienen el mismo centro se llaman concéntricas. Finalmente, combinando estas de-
finciones se tienen las circunferencias tangentes exteriormente y las tangentes interiormente.

Teorema: Dadas dos circunferencias de centros O1 y O2 que se cortan en dos puntos distintos
A y B, se cumple que O1 O2 ⊥ AB.

Teorema: Si dos circunferencias de centros O1 y O2 son tangentes en A, se cumple que O1 , A


y O2 están alineados.

Teorema:
16
También acá puede considerarse a las circunferencias tangentes como un caso especial de circunferencias secantes
en el cual los puntos de corte coinciden.

UES-FCNM-Escuela de Matemática-Profesorado en Matemática para Tercer Ciclo de Educación


Básica y Educación Media pág. 59
Geometrı́a I. 6 La Circunferencia

a) Dos circunferencias, una dentro de la otra, no tienen rectas tangentes en común.

b) Dos circunferencias tangentes interiormente tienen una recta tangente común.

c) Dos circunferencias secantes (en dos puntos distintos) tienen dos rectas tangentes en común.

d) Dos circunferencias tangentes exteriormente tienen tres rectas tangentes en común.

e) Dos circunferencias no secantes y tal que ninguna contiene a la otra, tienen cuatro rectas
tangentes en común.

Figura 67:

UES-FCNM-Escuela de Matemática-Profesorado en Matemática para Tercer Ciclo de Educación


Básica y Educación Media pág. 60
Geometrı́a I. 6 La Circunferencia

6.5. Problemas

1. Si el ∠M P Q = 20, determine el valor del ∠QON en la figura


adjunta.

2. Dado un ángulo inscrito BAC, y su ángulo central BOC, se sabe que ∠BAC +∠BOC = 180°.
Calcular el ∠OBC.

3. En la figura 68, BCDO es un rombo. Determine el valor del ángulo θ y la medida de las
diagonales de BCDO si el radio de la circunferencia mide 6.

Figura 68:

4. Un cuadrilátero cı́clico ABCD satisface ∠ABC = 2∠CDA = θ. Calcule θ.

5. En la figura 69. Calcule el valor del ∠P QR.

Figura 69:

UES-FCNM-Escuela de Matemática-Profesorado en Matemática para Tercer Ciclo de Educación


Básica y Educación Media pág. 61
Geometrı́a I. 6 La Circunferencia

6. En la figura adjunta, el ∠AF E = 100° y el ∠BCD =


150°. Calcule el ∠AGB.

7. Dado un ángulo ∠AOB, se trazan dos rectas l y m perpendiculares a los lados del ángulo
en A y B respectivamente. Si P es el punto de corte de l y m, demuestre que A, B, O, P se
ubican sobre una misma circunferencia.

8. En la figura 70 se ha tomado un punto C sobre la circunferencia de centro O; AC y BC


cortan a la segunda circunferencia en D y E respectivamente. Probar que OC ⊥ DE.

Figura 70:

UES-FCNM-Escuela de Matemática-Profesorado en Matemática para Tercer Ciclo de Educación


Básica y Educación Media pág. 62
Geometrı́a I. 6 La Circunferencia

9. (*) Dada la figura 71, demuestre que AB k A0 B 0 .

Figura 71:

10. En la figura 72 CR es una recta tangente en C, demuestre que AB k CR.

Figura 72:

11. Dos circunferencias Γ1 y Γ2 son tangentes (interior o exteriormente) en P (Ver figura 73). Dos
rectas que pasan por P cortan a Γ1 y Γ2 en A y C, y en B y D, respectivamente. Demuestre
que AB k CD.

Figura 73:

UES-FCNM-Escuela de Matemática-Profesorado en Matemática para Tercer Ciclo de Educación


Básica y Educación Media pág. 63
Geometrı́a I. 6 La Circunferencia

12. (*) Dos circunferencias de centros O1 y O2 son tangentes (interna o externamente) en un


punto P ; por este punto se traza una recta que corta nuevamente a la circunferencias en A
y B, respectivamente. Demuestre que AO1 k BO2 .

13. Dos circunferencias son tangentes externamente en el punto A. Una tangente exterior común
toca a una circunferencia en B y a la otra en C. Demostrar que ∠BAC = 90°.

14. En la figura 74, DE es tangente en D, y C es el punto medio del arco AD. Encuentre el
valor del ángulo seminscrito ADE.

Figura 74:

15. Determine el valor del ∠DCF , sabiendo BE es tangente en el punto D a la circunferencia


de centro O. Ver Figura 75.

Figura 75:

16. Si el ∠AEB = 30, ∠ADE = 20 y ∠ACE = 35, calcule el ∠AF B. Véase figura 76.

UES-FCNM-Escuela de Matemática-Profesorado en Matemática para Tercer Ciclo de Educación


Básica y Educación Media pág. 64
Geometrı́a I. 6 La Circunferencia

Figura 76:

17. Dada una circunferencia de diámetro BC, se toma un punto P en la prolongación de BC, y
se traza la tangente AP . Si AP = AB y O es el centro de la circunferencia, demuestre que
el 4AOC es equilátero.

18. (*) Dadas dos circunferencias una fuera de la otra, demuestre que las tangentes comunes
externas forman segmentos iguales; análogamente, las tangentes comunes internas forman
segmentos iguales.

19. (*) Teorema de Pithot. Demuestre que en todo cuadrilátero inscribible, la suma de lados
opuestos es igual.

20. (*) Teorema de Steiner. En todo cuadrilátero exinscrito a una circunferencia, la diferencia
de las longitudes de lados opuestos es igual.

21. En la figura 77, AB es una cuerda y por D se traza una recta tangente a la circunferencia
paralela a AB. Demuestre que CD es bisectriz del ∠ACB.

Figura 77:

22. (X OMCC - P2, Aarón) Sea ABCD un cuadrilátero concı́clico con diámetro AC, y sea O el
centro de su circunferencia. Se construyen los paralelogramos DAOE y BCOF . Demuestre
que si E y F están sobre la circunferencia entonces ABCD es rectángulo.

UES-FCNM-Escuela de Matemática-Profesorado en Matemática para Tercer Ciclo de Educación


Básica y Educación Media pág. 65
Geometrı́a I. 6 La Circunferencia

23. Cuatro cilindros de diámetro 1 están pegados apretadamente por


una cuerda muy fina, como en la figura adjunta. Demostrar que la
cuerda tine longitud 4 + π. Demostrar también que el área som-
breada entre los cilindros es 1 − π4 .

24. En la figura 78, ABCD es un trapecio isósceles con AB k CD y DA = BC = 2; tomando DA


y BC como diámetros, se construyen dos circunferencias tangentes. Si DC = 3AB, calcule
el área del trapecio.

Figura 78:

25. La figura 79 está formada por un paralelogramo y dos circunferencia tangentes entre sı́ y
tangentes a tres lados del paralelogramo. Sabiendo que el radio de las mismas mide la cuarta
parte del lado menor del paralelogramo, calcule la razón entre el lado mayor del paralelogramo
y el radio de las circunferencias.

Figura 79:

26. (*) Sea ABC un triángulo, y sean L y N las intersecciones de la bisectriz del ángulo A con
el lado BC y el circuncı́rculo de ABC respectivamente. Construimos la intersección M del
circuncı́rculo de ABL con el segmento AC. Prueba que los triángulos BM N y BM C tienen
la misma área.

27. (*) Sea AB el diámetro de una semicircunferencia. Se colocan los puntos M y K sobre la
semicircunferencia y sobre AB, respectivamente.17 Sea P el centro de la circunferencia que
pasa por A, K y M ; sea Q el centro de la circunferencia que pasa por B, K y M . Demuestre
que M P KQ es concı́clico.
17
M y K son distintos de A y B.

UES-FCNM-Escuela de Matemática-Profesorado en Matemática para Tercer Ciclo de Educación


Básica y Educación Media pág. 66
Geometrı́a I. 6 La Circunferencia

28. En la figura 80, ABCDEF es un hexágono regular y las circunferencias de centro en los
vértices son tangentes dos a dos. Si las circunferencias sobre los vértices B, D, F son iguales,
demuestre que las circunferencias restantes son iguales.

Figura 80:

29. (*) Las circunferencias Γ1 y Γ2 se cortan en los puntos A y B. Por el punto A se traza una recta
que corta nuevamente a las circunferencias Γ1 y Γ2 en los puntos C y D, respectivamente.
Por los puntos C y D se trazan tangentes a las circunferencias, las cuales se cortan en el
punto M . Demuestra que M CBD es cı́clico.
30. (*) El 4ABC cumple que ∠A = 90° y AB = AC. Se toma un punto E del segmento AB,
se construye interiormente un triángulo equilátero AEF . EF corta BC en I, y se construye
exteriormente un triángulo equilátero BIJ. Encuentre ∠EJB.
31. (*) En la figura 81, se sabe que ∠AO1 B − ∠AO2 B = 70◦ y además la tangente EB forma el
triángulo isósceles ABE, con AB = AE. Encuentre ∠EBC.

Figura 81:

32. (*) Dos circunferencias Γ1 y Γ2 se cortan en A y B. Una recta por A corta a Γ1 y Γ2 en C


y D, respectivamente, y la paralela a CD por B corta Γ1 y Γ2 en E y F , respectivamente.
Demuestre que 4CDB ≡ 4EAF .

UES-FCNM-Escuela de Matemática-Profesorado en Matemática para Tercer Ciclo de Educación


Básica y Educación Media pág. 67
Geometrı́a I. 6 La Circunferencia

33. (*) Sea P un punto exterior al cuadrado ABCD tal que ∠AP C = 90◦ , Q es la intersección
de AB y P C, y R el pie de la perpendicular por Q a CA. Demuestre que P , R y D están
alineados.

34. En la figura 82, ABCD es un trapecio rectángulo tal que la circunferencia de diámetro AB
(y centro O) es tangente a CD. Demostrar que O pertenece a la circunferencia de diámetro
CD y que esta circunferencia es tangente a BA.

Figura 82:

35. El 4ABC es rectángulo en C, la circunferencia de centro O es tangente a cada uno de los


lados del 4ABC en los puntos P , Q y R (como se muestra en la figura 83), y se cumple que
AP = 20 y BP = 6. Calcule OP .

Figura 83:

36. Los vértices A y B de un triángulo equilátero 4ABC están sobre una circunferencia de
radio 1 y el vértice C está en el interior de la circunferencia. Un punto D (distinto de B)
que esta en la circunferencia es tal que AD = AB. La recta DC corta por segunda vez a la
circunferencia en E. Encuentre la longitud del segmento CE. Ver figura 84.

37. En la figura 85 se muestran tres semicircunferencias, una de diámetro AB (de centro O y


radio r), otra de diámetro AO y la última de diámetro OB. Determine la razón entre el radio
de la circunferencia tangente a estas tres semicircunferencias y r.

38. El segmento AB es diámetro de un semicı́rculo con centro en O. Un cı́rculo con centro en P


es tangente a AB en O y también al semicı́rculo. Otro cı́rculo con centro en Q es tangente a
AB, al semicı́rculo y al cı́rculo de centro en P . Si AB = 2, ¿cuál es el radio del cı́rculo con
centro en Q?

UES-FCNM-Escuela de Matemática-Profesorado en Matemática para Tercer Ciclo de Educación


Básica y Educación Media pág. 68
Geometrı́a I. 6 La Circunferencia

Figura 84:

Figura 85:

Figura 86:

39. (*) (OIM 2002, P-4) En un triángulo escaleno ABC se traza la bisectriz interior BD, con D
sobre AC. Sean E y F puntos sobre la recta BD tales que (AE k CF ) ⊥ BD, y sea M el
punto sobre el lado BC tal que DM ⊥ BC. Demuestre que ∠EM D = ∠DM F .

40. (*) (OMCC 2003, P-2) Sea S una circunferencia y AB un diámetro de ella. Sea t la recta
tangente a S en B y considere dos puntos C y D en t tales que B este entre C y D. Sean E
y F las intersecciones de S con AC y AD y sean G y H las intersecciones de S con CF y
DE. Demuestre que AH = AG.

41. (*) (The 59th Romanian Mathematical Olympiad District Round) Considere un cuadrado
ABCD y un punto E sobre el lado AB. La diagonal AC corta al segmento DE en el punto
P . La perpendicular por P a DE corta al lado BC en F . Probar que EF = AE + CF .

42. (*) Teorema de Arquı́medes: En la figura 87, la región delimitada por tres semicircunfe-
rencias mutuamente tangentes, es conocida como cuchilla de zapatero o árbelos. Demostrar
que las circunferencias sombreadas son congruentes.

UES-FCNM-Escuela de Matemática-Profesorado en Matemática para Tercer Ciclo de Educación


Básica y Educación Media pág. 69
Geometrı́a I. 6 La Circunferencia

Figura 87: Teorema de Arquı́medes.

43. Demuestre que las rectas de Simson-Wallace de dos puntos diametralmente opuestos son
perpendiculares.

UES-FCNM-Escuela de Matemática-Profesorado en Matemática para Tercer Ciclo de Educación


Básica y Educación Media pág. 70
Geometrı́a I. 7 Semejanza de Triángulos.

7. Semejanza de Triángulos.
7.1. Proporcionalidad
Definición

1. Razón: se llama razón, al cociente de dos cantidades, expresadas en la misma magnitud, por
ejemplo ab .

2. Proporción: se llama proporción a la igualdad de dos razones. Por ejemplo ab = dc , 18 a los


términos a y d se les llama extremos y los términos b y c se les llama medios, al término d se
le llama cuarta proporcional entre a, b y c en este orden.

Propiedades de las proporciones:


a c
1. = si y sólo si a· c = b· d.
b d
a c b d a b
2. = si y sólo si = o = .
b d a c c d
a c a±b c±d
3. = si y sólo si = .
b d b d
a c a+b c+d
4. = si y sólo si = .
b d a−b c−d

7.2. Teorema de Thales


Definición
PA
1. Un punto P ∈ AB divide al segmento AB en una razón dada r, si PB
= r.

2. Sean AB y CD dos segmentos, y sean P ∈ AB y Q ∈ CD, decimos que P y Q dividen a


CQ
AB y CD en segementos proporcionales si PAPB = QD .

Figura 88:

18
En algunos textos de geometrı́a se utiliza la notación de proporción ası́ a : b :: c : d que se lee “a es a b como c
es a d”.

UES-FCNM-Escuela de Matemática-Profesorado en Matemática para Tercer Ciclo de Educación


Básica y Educación Media pág. 71
Geometrı́a I. 7 Semejanza de Triángulos.

Teorema de Thales. Si tres paralelas cortan a dos secantes entonces los segmentos que de-
terminan en ellas son proporcionales. 19

Antes de demostrar el Teorema de Thales, se enunciarán dos teoremas que a pesar de su apa-
rente sencillez es de mucha utilidad en problemas que involucran Áreas y Proporcionalidad.

Lema 1. Sea AB k CD. Demuestre que: (ABC) = (ABD).

Lema 2. Sea P un punto sobre el lado AB (o su prolongación) del 4ABC. Pruebe que:

AP (AP C)
=
PB (P BC)

A continuación se enuncian los pasos a seguir en la demostración del teorema de Thales.

Demostración. Sean AA0 , BB 0 y CC 0 rectas paralelas que cortan a dos secantes en los puntos
A, A0 , B, B 0 , C, C 0 respectivamente (ver figura 89).

Figura 89: Teorema de Thales

Pruebe que:
AB (ABB 0 )
1. =
BC (BCB 0 )
A0 B 0 (A0 B 0 B)
2. = .
B0C 0 (B 0 C 0 B)
3. (ABB 0 ) = (A0 B 0 B) y (BCB 0 ) = (B 0 C 0 B).

Con ayuda de las igualdades demostradas concluya que:

AB A0 B 0
= 0 0.
BC BC
Observación Importante: Utilice las propiedades de las proporciones para demostrar las equi-
valencias siguientes (interprételas geométricamente):

AB A0 B 0 AC A0 C 0 AC A0 C 0
= 0 0 ⇔ = 0 0 ⇔ = 0 0
BC BC AB AB BC BC
19
El teorema de Thales puede enunciarse de manera general como sigue: Si tres o más paralelas cortan a dos o
más secantes entonces los segmentos que determinan en ellas son proporcionales.

UES-FCNM-Escuela de Matemática-Profesorado en Matemática para Tercer Ciclo de Educación


Básica y Educación Media pág. 72
Geometrı́a I. 7 Semejanza de Triángulos.

Sea 4ABC un triángulo, sabemos que su área puede calcularse al multiplicar la longitud de
uno de sus lados por la longitud de la altura correspondiente a ese lado. Si denotamos por a la
longitud de un lado del 4ABC y ha la longitud de la altura correspondiente, el area del 4ABC
se denota por (ABC) y es igual a:
a · ha
2
Ahora estamos en condiciones de probar el siguiente lema:
Lema 3: Si dos triángulos tienen una misma altura entonces la razon entre sus areas es igual
a la razon de las bases donde se levanta la altura comun.
Demostracı́on:Sean 4ABC y 4A0 B 0 C 0 triángulos con alturas iguales h. Sean D y D0 los pies
de las alturas trazadas de los vertices A y A0 a los lados BC y B 0 C 0 respectivamente. Entonces:

BC · h 0 0 0 B0C 0 · h
(ABC) = (A B C ) =
2 2
La razón entre las áreas es:
BC·h
(ABC) 2 BC
= B 0 C 0 ·h
=
(A0 B 0 C 0 ) 2
B0C 0
Como queriamos demostrar.
El siguiente lema se puede probar de manera análoga al anterior:
Lema 4: Si dos triángulos tienen una base igual entonces la razón de sus áreas es igual a la
razón entre las alturas que se levantan sobre la base igual.
Corolario (Teorema de Thales en el triángulo). Toda recta paralela a un lado de un triángulo
y que corte a los otros dos lados, divide a estos lados en segmentos proporcionales.

Recı́proco del Teorema de Thales. Si tres rectas cortan a dos secantes en segmentos pro-
porcionales y dos de estas rectas son paralelas entonces las tres rectas son paralelas.

Demostración. Sean AA0 , BB 0 y CC 0 rectas que cortan a dos secantes en los puntos A, A0 ,
0 0 0 0 AB A0 B 0
B, B , C, C respectivamente, tales que AA k CC y = 0 0 . Por el punto B tracemos
0 0 0
BC BC
una recta paralela a AA , la cual interseca a A C en el punto D (ver figura 90). Entonces, por el
AB A0 D A0 B 0 A0 D
Teorema de Thales se tiene que: = 0
. De donde, 0C 0
= 0
, ası́ por las propiedades
BC DC B DC
A0 C 0 A0 C 0
de las proporciones 0 0 = 0
, por lo que B 0 C 0 = B 0 D + DC 0 = DC 0 y por tanto B 0 D = 0, o
B C DC
equivalentemente B 0 = D y por lo tanto, BB 0 k AA0 .

Figura 90: Recı́proco del Teorema de Thales

UES-FCNM-Escuela de Matemática-Profesorado en Matemática para Tercer Ciclo de Educación


Básica y Educación Media pág. 73
Geometrı́a I. 7 Semejanza de Triángulos.

Corolario (Recı́proco del Teorema de Thales en el triángulo.) Si una recta intercepta


dos lados de un triángulo en segmentos proporcionales entonces la recta es paralela al tercer lado
del triángulo.
Teorema:Sea 4ABC un triangulo y sean D y E puntos en los lados AB y AC respectivamente.
Si se cumple que:
AB AC
=
AD AE
entonces DE es paralela a BC.
Supongamos que, por el contrario DE no es paralela a BC. Sea C 0 un punto en AC, distinto
0
de C, tal que BC 0 es paralela a DE, entonces por el teorema de Thales, AD
AB
= AC
AE
. Pero ademas,
AB AC
se cumple por hipotesis del problema, que AD = AE . Podemos entonces decir que
AB · AE
AC = = AC 0
AD

AC = AC 0
Por tanto, los puntos C y C 0 deben ser iguales y DE es paralela a BC.

7.3. Criterios de Semejanza de Triángulos


Triángulos semejantes. Decimos que el 4ABC es semejante al 4A0 B 0 C 0 (Ver figura 91), lo
cual denotamos ası́ ABC ∼ A0 B 0 C 0 , si:
AB AC BC
0 0
= 0 0 = 0 0
AB AC BC
y
∠BAC = ∠B 0 A0 C 0 , ∠ABC = ∠A0 B 0 C 0 , ∠ACB = ∠A0 C 0 B 0 .

Figura 91: Definición de Semajanza de Triángulos.

En los tres teoremas que se muestran a continuación (los cuales son una consecuencia directa
del Teorema de Thales) se establecen las condiciones mı́nimas para demostrar que dos triángulos
son semejantes, a los cuales denominaremos: Criterios de Semejanza de Triángulos.

Primer criterio de semejanza de triángulos: Angulo-Angulo A-A. Si dos ángulos de


un triángulo son congruentes con dos ángulos de otro triángulo, entonces los dos triángulos son
semejantes.

UES-FCNM-Escuela de Matemática-Profesorado en Matemática para Tercer Ciclo de Educación


Básica y Educación Media pág. 74
Geometrı́a I. 7 Semejanza de Triángulos.

Demostración. Supongamos que en el 4ABC y 4A0 B 0 C 0 se tiene que ∠ABC = ∠A0 B 0 C 0 y


∠ACB = ∠A0 C 0 B 0 , entonces ∠BAC = ∠B 0 A0 C 0 (Por la suma de ángulos internos en un triángulo).
Sea D ∈ AB y E ∈ AC tales que AD = A0 B 0 y AE = A0 C 0 , dado que ∠DAE = ∠BAC =
∠B 0 A0 C 0 , se sigue por L-A-L que 4ADE = 4A0 B 0 C 0 , por consiguiente ∠ADE = ∠A0 B 0 C 0 =
∠ABC, de donde DE k BC (por ser iguales los ángulos correspondientes) y por el teorema de
Thales
AB AC
=
AD AE
y por consiguiente
AB AC
0 0
= 0 0 (2)
AB AC
Sea F ∈ BC tal que DF k AC, entonces F C = DE = B 0 C 0 (porque DECF es paralelogramo
y por ser 4ADE = 4A0 B 0 C 0 ) y por el teorema de Thales
BA BC
=
DA FC
o lo que es lo mismo
AB BC
0 0
= 0 0 (3)
AB BC
Luego, de (1) y (2) se tiene que:
AB AC BC
0 0
= 0 0 = 0 0.
AB AC BC
Ası́, se ha demostrado que los tres pares de ángulos son congruentes y los tres pares de lados son
proporcionales, por lo tanto, 4ABC ∼ 4A0 B 0 C 0 .

Segundo criterio de semejanza de triángulos: L-A-L. Si un ángulo de un triángulo es


congruente con otro ángulo de otro triángulo y los lados que comprenden al ángulo en el primer
triángulo son respectivamente proporcionales a los lados que comprende al ángulo en el segundo
triángulo, entonces los dos triángulos son semejantes.
AB AC
Demostración. Suponga que el ∠BAC = ∠B 0 A0 C 0 y que 0 0 = 0 0 . Considere los
AB AC
puntos D y E, como en la demostración del teorema anterior. Entonces por el criterio L-A-L,
4ADE = 4A0 B 0 C 0 , de lo cual se deduce que ∠ADE = ∠A0 B 0 C 0 . Por otra parte tenemos que:
AB AC
= , y al aplicar el recı́proco del teorema de Thales, se puede afirmar que DE k BC, de
AD AE
lo cual a su vez se deduce que ∠ADE = ∠ABC, por ángulos correspondientes entre paralelas.
Finalmente por transitividad se concluye que ∠ABC = ∠A0 B 0 C 0 . Por lo tanto, 4ABC ∼ 4A0 B 0 C 0
(Por el criterio A-A.)

Tercer criterio de semejanza de triángulos: L-L-L. Si los tres lados de un triángulo son
respectivamente proporcionales a los tres lados de otro triángulo, entonces los dos triángulos son
semejantes.
AB AC BC
Demostración. Por hipótesis se tiene que: 0 0 = 0 0 = 0 0 y como antes sean D y E
AB AC BC
puntos sobre AB y AC respectivamente tales que AD = A0 B 0 y AE = A0 C 0 . Entonces por el
recı́proco del teorema de Thales se tiene que DE k BC y por consiguiente el ∠ABC = ∠ADE
AB BC
y el ∠ACB = ∠AED, de donde 4ABC ∼ 4ADE (por el criterio A-A). Por ende = ,
AD DE

UES-FCNM-Escuela de Matemática-Profesorado en Matemática para Tercer Ciclo de Educación


Básica y Educación Media pág. 75
Geometrı́a I. 7 Semejanza de Triángulos.

BC BC
luego por transitividad = 0 0 , de donde DE = B 0 C 0 . En consecuencia 4ADE = 4A0 B 0 C 0
DE BC
(por el criterio L-L-L), de lo cual se sigue que ∠A0 B 0 C 0 = ∠ADE y ∠A0 C 0 B 0 = ∠AED, y por
transitividad ∠A0 B 0 C 0 = ∠ABC y ∠A0 C 0 B 0 = ∠ACB =. Por lo tanto, 4A0 B 0 C 0 ∼ 4ABC (Por
el criterio A-A.)

7.4. Potencia de Punto


Proposición.
Si dos cuerdas AB y CD de una circunferencia se intersectan en un punto P , entonces P A·P B =
P C · P D.

Demostración. Si P es un punto sobre la circunferencia está claro que ambos productos


son 0 y por tanto iguales. Si P es un punto dentro de la circunferencia entonces el cuadrilátero
ADBC es cı́clico con los ángulos ∠BAD = ∠BCD y ∠ADC = ∠ABC y por tanto los triángulos
4AP D y 4BP C y por tanto se cumple las proporciones PP CA = PP DB
y despejando obtenemos que
P A · P B = P C · P D.

Si P es un punto fuera de la circunferencia el cuadrilátero ABDC es ciclico y los ángulos ∠CAP


y ∠BDP son iguales y los triángulos 4P AC y 4P BD son triángulos semejantes con lo que se
cumple de nuevo en este caso que PP CA = PP D
B
y por tanto, P A · P B = P C · P D.

Proposición 2. Si A, B y C son puntos sobre una circunferencia y si la tangente en C, inter-


secta en un punto P a la prolongación de la cuerda AB, entonces P C 2 = P A · P B.

Demostración. Sabemos que un ángulo seminscrito es igual a un ángulo inscrito que sostenga
el mismo arco asi pues, el ángulo ∠P CA = ∠CBP y dado que, ∠CP A = ∠CP B los triángulos
4P CA y 4BCP son semejantes y se cumple la razón PP CA = PP B
C
y por tanto, P C 2 = P A · P B.

Ahora veremos los inversos de las proposiciones anteriores.

Proposición 3. a. Si AB, CD son dos segmentos que se intersectan en P de manera que, como
segmenteos dirigidos, P A · P B = P C · P D entonces A,B, C y D se encuentran sobre una circun-
ferencia.

b. Si A, B, C, y P son puntos tales que P, A y B están alineados y P C 2 = P A.P B, entonces


P C es tangente en C al circuncı́rculo del triángulos 4ABC.

Demostración. Denotemos por Γ la circunferencia que inscribe al triángulo ABC.

a. Supongamos que D no está sobre la circunferencia y sea D0 la intersección de P C con Γ.


Por la proposición 1 P A · P B = P C · P D0 y dado que la hipótesis dice que P A · P B = P C · P D,
tenemos que P D = P D0 , por lo que D y D0 son iguales. Contradicción. Por tanto, D debe estar
sobre la circunferencia.

b. Sea C 0 la otra intersección de P C con Γ. Por la primera proposición, P A · P B = P C · P C 0


y como por hipótesis P A · P B = P C 2 , obtenemos que P C 0 = P C y entonces C coincide con C 0 .

UES-FCNM-Escuela de Matemática-Profesorado en Matemática para Tercer Ciclo de Educación


Básica y Educación Media pág. 76
Geometrı́a I. 7 Semejanza de Triángulos.

Podemos concliur que P C es tangente a Γ en C.

Para cualquier punto P , una circunferencia C y cualquier recta trazada por P que corte a C
en puntos A y B (A y B pueden ser iguales) que: el producto P A · P B es constante y se denomina
la potencia de punto del punto P con respecto a C.

Otra forma de calcular la potencia de un punto se comentara a continuación. Si P es exterior


a C y P C es tangente a C por P , la potencia es P C 2 . Si O es el centro de la circunferencia C y
radio r, tenemos que, por el teorema de Pitágoras: P C 2 = P O2 − r2 .

Ahora bien, si P es un punto dentro de la circunferencia y que está sobre un diámetro AB,
podemos calcular la potencia como P A · P B. Pero vemos que P A lo podemos escribir como r − P O
y P B como r + P O, o viceversa, dependiendo de si P esta mas cerca de A o de B. De ahı́ que
P A · P B = (r − P O)(r + P O) = r2 − P O2 . Si un punto esta sobre la circunferencia sabemos que
su potencia es cero, lo cual también es descrito por r2 − P O2 .

Para teminar podemos decir que la potencia de punto de P con respecto a la circunferencia
C = (O, r) es P O2 − r2 . Además, la potencia es positiva, cero o negativa, dependiendo si P se
encuentra fuera, sobre o dentro de la circunferencia.

Eje Radical
Estudiaremos una familia muy importante de circunferencias. La propiedad que define a la
familia es que existe un punto en el plano tal que la potencia de ese punto con respecto a todas
las circunferencias de la familia es la misma, es decir, que el valor de la potencia es constante.

Definición: El eje radical de dos circunferencias es el lugar geométrico de todos los puntos
P ∈ E2 cuyas potencias con respecto a las dos circunferencias son iguales.

El siguiente teorema nos da la forma de obtener el eje radical de dos circunferencias dependien-
do de la posición de estas. el único caso en donde no existe el eje radical de dos circunferencias es
cuando una está contenida en la otra.

Teorema: Sean C1 y C2 dos circunferencias. Entonces se cumple una de las siguientes propie-
dades:

1. Si C1 y C2 se intersecan, entonces su eje radical es la lı́nea que pasa por los puntos de
intersección.

2. Si las dos circunferencias C1 y C2 son tangentes, entonces su eje radical es la lı́nea tangente
común.

3. Sean C1 y C2 circunferencias ajenas y k, l, m, n son los ejes radicales de C1 y C3 , C1 y C4 ,


C2 y C3 , C2 y C4 respectivamente para cualesquiera dos circunferencias C3 y C4 en E2 que
cortan a C1 y C2 . Sea P el punto de intersección de n y l y sea Q el punto de intersección
de m y k. Entonces el eje radical de C1 y C2 es la lı́nea que pasa por P y Q.

UES-FCNM-Escuela de Matemática-Profesorado en Matemática para Tercer Ciclo de Educación


Básica y Educación Media pág. 77
Geometrı́a I. 7 Semejanza de Triángulos.

Eje radical de C1 y C2
C3 l
k n

P
C1 C2
Q

m
C4

Figura 92: Eje radical de dos circunferencias ajenas.

Corolario: Sean C1 , C2 y C3 tres circunferencias en E2 cuyos centros no son colineales. En-


tonces, los tres ejes radicales son concurrentes en un punto llamado el centro radical de las circun-
ferencias. Observacion

1. El eje radical de dos circunferencias es ortogonal a la lı́nea de los centros.

2. El eje radical de dos circunferencias, no contenidas una en la otra, es la lı́nea que pasa por
los puntos medios de los segmentos determinados por los puntos de contacto de las tangentes
comunes a las circunferencias.

Un conjunto de circunferencias se dice que son una familia cuando cumplen una o varias pro-
piedades, a continuación definimos una de ellas.

7.5. Problemas
a b c
1. Los puntos A, B, C y D forman una cuaterna armónica. Si AB
+ AD
= AC
; hallar a + b + c.

2. Dados los puntos A, B, C, D y E son colineales y consecutivos de modo que AB > BC


yBD > DE. Se sabe que√ AB y BD son secciones áureas de AC y BE respectivamente. Si
3− 5
BC = 2CD y AE = 2 ; calcular AC.

3. Sean AB y CD las bases del trapecio ABCD, cuyas diagonales se intersecan en E perpen-
dicularmente. Si AD = 13, AE = 12 y CE = 4 encuentre las longitudes de CD y AB.

4. En la figura 93, el 4ABC es equilátero, sus lados tienen longitud 3 y P A es paralela a BC.
Si P Q = QR = RS, encontrar la longitud de CS.

5. Sea ABCD un trapecio de bases BC y AD, sus diagonales se cortan en E. Si BE = 3,


ED = 4 y CE = 2, determine la medida de AE.

6. Las bases de un trapecio miden 3 y 5, y si su altura mide 4. Encontrar la distancia desde el


punto de corte de las diagonales hasta la base mayor.

UES-FCNM-Escuela de Matemática-Profesorado en Matemática para Tercer Ciclo de Educación


Básica y Educación Media pág. 78
Geometrı́a I. 7 Semejanza de Triángulos.

Figura 93:

7. En la figura adjunta, el 4ABC es rectángulo en A y el 4ADB


es rectángulo en D. El punto E es el punto de intersección de
los segmentos AD y BC. Si AC = 15, AD = 16 y BD = 12,
calcule el área del 4ABE.

8. El 4ABC es rectángulo en B. Se dibuja un rectángulo BEDF con D sobre la hipotenusa,


BC 1
E y F sobre BC y AB, respectivamente. Si AB = 1, demuestre que BE = 1−DE .
9. Considérese los puntos A, B, C y D tales que A y B están sobre el segmento OC y OD respec-
tivamente, donde O es el centro de la circunferencia de radio r (Ver figura 94).
 Si OA· OC =
2
r
r2 = OB· OD, demuestre que el 4AOB ' 4DOC y que CD = AB.20
OA· OB

Figura 94:

10. Sobre la circunferencia de centro O, se trazan los diámetros AB y CD tales que AB ⊥ CD.
Sea P un punto sobre el arco CBD y Q el punto de intersección de las cuerdas AP y CD.
Si DO = 1, demuestre que AP · AQ = 2.
11. Un segmento de recta AB es divido por los puntos interiores K y L de manera que AL2 =
AK· AB. Sea P un punto exterior al segmento AB tal que AP = AL. Pruebe que ∠KP L =
∠LP B. Figura 95.
20
La medida del segmento CD se denomina Distancia Inversa.

UES-FCNM-Escuela de Matemática-Profesorado en Matemática para Tercer Ciclo de Educación


Básica y Educación Media pág. 79
Geometrı́a I. 7 Semejanza de Triángulos.

Figura 95:

12. En la figura 96, AB y AC son tangentes a la circunferencia, y CE ⊥ BD, siendo BD un


diámetro. Probar que BE ·BO = AB ·CE.

Figura 96:

1 1 1
13. Demostrar que + = si se cumple que AX k BY k CZ. (Ver figura 97.)
AX BY CZ

Figura 97:

14. En la figura 98, el 4ABC es rectángulo. Se construyen exteriormente los cuadrados ABEF
y BCP Q. Demostrar que BM = BN .

UES-FCNM-Escuela de Matemática-Profesorado en Matemática para Tercer Ciclo de Educación


Básica y Educación Media pág. 80
Geometrı́a I. 7 Semejanza de Triángulos.

Figura 98: .

15. Sean O, P y R los centros de las tres circunferencias. Si OR = r y Q es la intersección de


P O con la circunferencia de centro R, demuestre que OP · OQ = r2 . Ver figura 99.

Figura 99:

16. Si en un triángulo rectángulo se traza la altura correspondiente a la hipotenusa, entonces:

a) Los dos nuevos triángulos que resultan, son semejantes entre si y semejantes al triángulo
original.
21
b) La altura es media proporcional entre los segmentos que ella determina sobre la
hipotenusa.
c) Cada cateto es media proporcional entre la hipotenusa y la proyección del cateto sobre
la hipotenusa.
d ) Demuestre el teorema de Pitágoras.

17. Si dos triángulos tienen sus lados respectivamente paralelos o respectivamente perpendicu-
lares, entonces los dos triángulos son semejantes.
21
Si b es una magnitud tal que ab = cb , entonces decimos que b es media proporcional entre a y c,o de manera
equivalente: b es media proporcional entre a y c si y solo si b2 = a· c.

UES-FCNM-Escuela de Matemática-Profesorado en Matemática para Tercer Ciclo de Educación


Básica y Educación Media pág. 81
Geometrı́a I. 7 Semejanza de Triángulos.

18. Sean ABC y A0 B 0 C 0 dos triángulos semejantes con AAB BC CA


0 B 0 = B 0 C 0 = C 0 A0 = k. Demuestre que:

la razón entre los perı́metros de los triángulos es k y que la razón entre sus áreas es k 2 .

19. Teorema de Menelao. Dado el 4ABC, sea P un punto sobre la recta AB, Q un punto
sobre la recta BC, R un punto sobre la recta CA. Si los puntos P , Q, R están alineados
AP BQ CR
entonces = 1.
P B QC RA

Figura 100: Teorema de Menelao.

Para demostrar este teorema, sea W un punto sobre la recta P QR tal que BW k AC:

a) Demuestre que los triángulos AP R y BP W son semejantes.


b) Demuestre que los triángulos CQR y BQW son semejantes.
AP BQ CR
c) De los literales a) y b) deduzca que = 1.
P B QC RA

20. Teorema de Ceva. Dado el 4ABC, sea P


un punto sobre el recta AB, Q un punto so-
bre la recta BC y R un punto sobre la recta
CA. Si las rectas AQ, CP , BR concurren,
AP BQ CR
entonces = 1.
P B QC RA

Para demostrar este teorema, sean W y V los puntos de intersección de la recta que pasa
por B paralela a AC, con las rectas CP y AQ, respectivamente.

a) Demuestre que 4AP C ∼ 4BP W y que 4AQC ∼ 4V QB.


b) Demuestre que 4BW O ∼ 4RCO y que 4BV O ∼ 4RAO.
AP BQ CR
c) Utilice los literales a) y b) para probar que = 1.
P B QC RA

UES-FCNM-Escuela de Matemática-Profesorado en Matemática para Tercer Ciclo de Educación


Básica y Educación Media pág. 82
Geometrı́a I. 7 Semejanza de Triángulos.

21. Sea ABC un triángulo, con D sobre AB, E sobre AC y F sobre BC, tal que DE es paralelo
a BC, Demuestre que BF = F C

22. Sea ABC un triángulo, con D sobre AB, con E sobre BC y F sobre AC, tal que AD = 2BD
y F A = 2CF . Demuestre que E es punto medio de B.

23. Sea Γ una circunferencia y dado un triángulo ABC, sean L, L0 , M, M 0 , N, N 0 los puntos de
corte de la circunferencia con el triángulo , sobre los lados BC, AC, AB respectivamente.
Demostrar que si AL, BM, CN concurren, entonces AL0 , BM 0 , CN 0 concurren.

24. En el triángulo ABC, rectángulo en A , se consideran las circunferencias inscritas y circuns-


critas. La recta AM es tangente a la circunferencia circunscrita en el punto A (M es punto
de BC). S y R son los puntos de tangencia de la circunferencia inscrita con los catetos AC y
AB, respectivamente. La recta RS corta a la recta BC en N . Las rectas AM y SR se cortan
en U . Demostrar que el triángulo U M N es isósceles.

25. Demuestre que si ABC es un triángulo y AA0 es su bisectriz externa (con A0 sobre BC)
0
entonces BA
A0 C
= AB
AC

26. Demuestre que si ABC es un triángulo y suponga que las bisectrices internas de B y C
cortan a CA y AB en b0 y C 0 respectivamente y que la bisectriz externa de A corta a BC en
A0 . Demuestre que A0 , B 0 , C 0 son colineales.

27. Si dos cuerdas se interceptan en el interior de una circunferencia entonces el producto de las
medidas de los segmentos determinados por el punto de intersección en una de las cuerdas
es igual al producto de las medidas de los segmentos determinados en la otra cuerda.

28. Si dos segmentos se interceptan en un punto que esta en el interior de los dos segmentos
y el producto de las medidas de los segmentos determinados por el punto de intersección
en el primer segmento es igual al producto de las medidas de los segmentos determinados
por el punto en el segundo segmento,entonces los extremos de los segmentos están sobre una
circunferencia.

29. Si desde un punto P exterior a una circunferencia se trazan dos semirrectas secantes que
cortan a la circunferencia en los puntos A, B y C, D respectivamente, entonces P A· P B =
P C· P D.

30. Si desde un punto P se trazan dos semirrectas con los puntos A, B sobre una y los puntos C,
D sobre la otra, tales que P A· P B = P C· P D, entonces los puntos A, B, C, D están sobre
una circunferencia.

31. Si desde un punto exterior a una circunferencia se trazan dos semirrectas, una tangente y la
otra secante, entonces el segmento entre el punto y el punto de tangencia es media propor-

UES-FCNM-Escuela de Matemática-Profesorado en Matemática para Tercer Ciclo de Educación


Básica y Educación Media pág. 83
Geometrı́a I. 7 Semejanza de Triángulos.

cional entre los segmentos determinados entre el punto exterior y los puntos de intersección
de la secante con la circunferencia. 22

32. Si P es un punto sobre el mismo plano que una circunferencia de centro O y radio r, y d es
la distancia del punto P al centro O de la circunferencia, demuestre que:

a) Si P está en el interior de la circunferencia, entonces la potencia de P es r2 − d2 .


b) Si P está en el exterior de la circunferencia, entonces la potencia de P es d2 − r2 .
c) Si P está sobre de la circunferencia, entonces la potencia de P es cero.

33. En un triángulo ABC los puntos D y E están en los lados AC y AB de forma tal que los
ángulos ∠ADE = ∠ABD. Si AE = 2 y BE = 3 encuentre AD.

34. El cuadrilátero ABCD es cı́clico y la intersección de los lados AC y BD es P . Halle BP si


P C = 4, DP = 2 y AP = 8.

35. En un cuadrilátero ABCD los ángulos en ∠A y en ∠C son rectos. Si los lados AB = 3,


AD = 4 y DP = 2, hallar hallar AP · P C.

36. Por un punto P sobre la cuerda común AB de dos circunferencias que se intersectan se tra-
zan las cuerdas KM sobre el primer cı́rculo y LN sobre el segundo cı́rculo. Pruebe que el
cuadrilátero KLM N es concı́clico.

37. En el triángulo ABC con circuncentro O, las alturas BE y CF se cortan en H. Las rectas
OB y HC se cortan en P . Si HP = 3, OP = 2 y P C = 5, ¿cuánto vale OB?

38. La recta OA es tangente a una circunferencia en el punto A y la cuerda BC es paralela a


OA. Las rectas OB y OC intersectan a la circunferencia por segunda vez en los puntos K y
L, respectivamente. Pruebe que la recta KL divide al segmento OA por la mitad.

39. Dada una circunferencia S, los puntos A y B sobre esta y C en la cuerda AB. Para cualquier
circunfenrencia S 0 tangente a la cuerda AB en el punto C y que intersecta a S en P y Q,
considere el punto de intersección M de las rectas AB y P Q. Pruebe que la posición de M
no depende de la eleccción de S 0

40. Desde A se trazan una recta tangente a la circunferencia S en C y una recta que corta a S
en los puntos ByD. Sea E otro punto en la circunferencia tal que CE es tangente a AD. Si
CA = 6, AB = 12 y CE = 4BD hallar CE y BD. Si P es el punto de intersección de CD
con BE cual es la potencia de punto de P con respecto a S.

22
Los problemas anteriores nos permite establecer la siguiente definición de Potencia de un punto con respecto
a una circunferencia: La potencia de un punto P con respecto a una circunferencia de centro O y radio r es el
producto P A· P B, donde A y B son los puntos de intersección de la circunferencia con una recta que pasa por P .

UES-FCNM-Escuela de Matemática-Profesorado en Matemática para Tercer Ciclo de Educación


Básica y Educación Media pág. 84
Geometrı́a I. 7 Semejanza de Triángulos.

41. Dos circunferencias se intersectan en los puntos A y B y sea M N su tangente comun. Pruebe
que la recta AB divide a M N por la mitad.

42. Pruebe que el eje radical de dos circunferencias es el lugar geométrico de los puntos P tales
que al trazar las tangentes a las circunferencias la distancia de P a ambos puntos de tangen-
cia es la misma.

43. A partir de un punto P exterior a una circunferencia de centro O se trazan las tangentes

P Q y la secante P BA, tal que P Q = AB = 2, si el radio de la circunferencia mide 1 + 5 ,
hallar ∠BOP .

44. Dado el 4ABC se construye un cuadrado P QRS con P en AB, Q en AC, R y S en BC.
Sea H el pie de la altura desde A hacia BC. Demuestre que:
1 1 1
a) = +
PQ AH BC
b) (ABC) = 2(P QRS) si y sólo si AH = BC.

45. Sea P un punto en el interior del 4ABC. Se trazan por P las paralelas a los lados del
triángulo, que queda dividido en tres triángulos y tres paralelogramos. Si las áreas de los tres
triángulos de la subdivisión son, en algún orden, 9, 16 y 25, hallar el área del 4ABC.

46. Las tres circunferencias de la figura 101 tienen el mismo radio r, sus centros son colineales y
la circunferencia de centro O2 es tangente a las otras dos. Por A se traza una tangente a la
circunferencia de centro O3 . Obtenga el valor del segmento BC en función de r.

Figura 101:

47. Sea ABCD un rombo, con AC = 6 y BD = 8. Se construyen exteriormente los cuadrados


ADEF y CDHG, cuyos centros son O1 y O2 , respectivamente (Vea figura 102). Calcular la
medida del segmento O1 O2 .

UES-FCNM-Escuela de Matemática-Profesorado en Matemática para Tercer Ciclo de Educación


Básica y Educación Media pág. 85
Geometrı́a I. 7 Semejanza de Triángulos.

Figura 102:

48. Sea ABCD un cuadrado con P y Q sobre AB y BC tales que BP = BQ. Sea H el pie de
la perpendicular de B a P C. Demuestre que DHQ = 90°.
49. Alrededor de una circunferencia se construyen diez circunferencias tangentes a la original
y tangentes entre sı́ (Véase figura 103). Demuestre que la suma de las áreas de las diez
circunferencias es el doble del área de la circunferencia mayor.

Figura 103:

50. En un 4ABC el ∠CAB = 120. Encuentre la medida de la bisectriz interna del ∠CAB en
función de los lados adyacentes.
51. El 4ABC tiene lados de 13, 14 y 15 unidades. El 4A0 B 0 C 0 está dentro del 4ABC con
lados paralelos a los de éste y a 2 unidades de distancia de los lados del mismo. Calcule
(ABC) − (A0 B 0 C 0 ).

UES-FCNM-Escuela de Matemática-Profesorado en Matemática para Tercer Ciclo de Educación


Básica y Educación Media pág. 86
Geometrı́a I. 8 ISOMETRÍAS

8. ISOMETRÍAS
Consideremos el conjunto P de puntos del plano y Φ : P −→ P una aplicación. Se dice que
Φ es una ISOMETRÍA si tiene la propiedad de conservar la distancia entre puntos; es decir, si
dos puntos P y Q guardan entre sı́ una distancia l, la distancia que guardan sus imágenes por
la transformación es también l. Ası́, d(P, Q) = d(Φ(P ), Φ(Q)) = l. En este curso estudiaremos
las traslaciones, las rotaciones, simetrı́as centrales, simetrı́as ortogonales o reflexiones; que tienen
todas ellas la propiedad de conservar distancias.

8.1. Traslaciones.
Definición: Sea →

u un vector, se llama traslación del vector →
−u a la asociación de los puntos M
0
− −−→ 0 →

y M del plano tales que: M M = u .

− 0 t→

u
M0
En general se denota t−
→u a la traslación del vector u , y escribimos: M = t− →uM o M − →

8.1.1. Propiedades de las Traslaciones.


Las traslaciones poseen las siguientes propiedades:

La imagen de una recta, es otra recta paralela a ella.

La imagen de un segmento, es otro segmento de igual longitud.

La imagen de una circunferencia de centro O, es otra circunferencia del mismo radio, cuyo
centro es la imagen de O por la traslación.

Conservación del alineamiento de puntos.


Dado que la imagen de una recta es otra recta, también se cumple que si tres puntos A, B, C
están alineados, las imágenes respectivas A0 , B 0 , C 0 están también alineados.

Conservación del paralelismo.


Si d1 y d2 son dos rectas paralelas, las imágenes respectivas d01 y d02 son paralelas también.
De esta propiedad es fácil deducir que: “La imagen de un paralelogramo, es otro paralelo-
gramo ”.

Conservación de las distancias y las áreas.


La imagen de un segmento, es también un segmento de la misma longitud.
La imagen D0 de una superficie D, tiene la misma área que D.

Conservación del punto medio de un segmento.


Sea I el punto medio del segmento AB, si A0 B 0 es la imagen del segmento AB, entonces I 0
es la imagen de I, además I 0 es el punto medio del segmento A0 B 0 .

Conservación de la medida de ángulo y de la ortogonalidad.


La imagen x\0 A0 y 0 del ángulo xAy
d tiene la misma medida que xAy.
d
En particular cuando dos rectas d1 y d2 son perpendiculares sus respectivas imágnes d01 y d02
son también perpendiculares.

UES-FCNM-Escuela de Matemática-Profesorado en Matemática para Tercer Ciclo de Educación


Básica y Educación Media pág. 87
Geometrı́a I. 8 ISOMETRÍAS

−→
Ejemplo 8.1 A, B, C son tres puntos tales que C es la imagen de B por la traslación de AB.
Haga una figura de esta situación y responda ¿Cuál es el punto medio de AC?
−→
Prueba 1 → −u = AB luego por definición tu (A) = B, y de acuerdo a las propiedades de las tras-
laciones la imagen de AB es otro segmento de la misma longitud, y como AB = BC, entonces B
es el punto medio de AC.

A u B C

Ejemplo 8.2 4ABC es un triángulo cualquiera. Denotaremos por B 0 a la imagen de B por la


simetrı́a central de centro A. Trazamos la recta d paralela a AB y que pasa por C, y la recta ∆
paralela a BC y que pasa por A. Las rectas d y ∆ se cortan en C 0 . Demostrar que el triángulo
4B 0 AC 0 es la imagen del triángulo 4ABC por traslación.

Prueba 2 De acuerdo a la forma del 4ABC (Ver figura), transformamos el 4ABC en el 4B 0 AC 0


−→ −−→ −−→
por la traslación del vector BA hacia A0 B 0 o CC 0 . Por lo cual, será suficiente demostrar que
tu (A) = B 0 , tu (B) = A y tu (C) = C 0 .
tu (B) = A, este resultado se obtiene de la definición de traslación del vector →

u.

tu (A) = B 0 , en efecto, B 0 es la imagen de B, por la simetrı́a central de centro A. Como A


−→ −−→ −−→ −
es el punto medio de BB 0 , se tiene que BA = AB 0 . Es decir AB 0 = → u y esto por definición
0
es tu (A) = B .

tu (C) = C 0 , en efecto, los lados opuestos del cuadrilátero AC 0 CB son dos a dos paralelos, es
−−→ −→ −−→ −
decir se trata de un paralelogramo, esto es: CC 0 = BA =⇒ CC 0 = → u , lo cual significa que
0
tu (C) = C .

B0

A C0

B C

Ejemplo 8.3 Dado el 4ABC sea H su ortocentro. BCDE es un rectángulo construido exterior-
mente. Se trazan, por D la perpendicular a AB, y por E la perpendicular a AC. Estas dos rectas

− −−→
se cortan en I. Utilice la traslación t−
u del vector u = EB para demostrar que A, H e I están

alineados.
−−→
Prueba 3 Dado → −
u = EB, por definición tu (E) = B, y dado que BCDE es un rectángulo también
se cumple tu (D) = C. Como la imagen por una traslación de una recta es otra recta paralela a la
primera, la recta BM es la imagen por → −
u de la recta ER, dado que al ser ambas perpendiculares

UES-FCNM-Escuela de Matemática-Profesorado en Matemática para Tercer Ciclo de Educación


Básica y Educación Media pág. 88
Geometrı́a I. 8 ISOMETRÍAS

a AC resulta inmediato que ER k BM . Análogamente, la imagen de la recta DS por →



u es la recta
CP .

La intersección de ER y DS es I, como la traslación es biyectiva (en el sentido de que es


punto a punto) ésta lleva intersecciones de rectas a intersecciones de rectas, luego t−
u (I) = H y

−→ → −
esto implica que IH ⊥ BC (dado que IH = u por definición de traslación, y este último vector
evidentemente es perpendicular a BC dada la forma cómo se definió). Como H es el ortocentro
del 4ABC, AH ⊥ BC. Ası́ IH k AH (ambas son perpendiculares a BC) y esto obliga a que A,
H, I estén alineados.

P M
H
R
S

B C

E D

UES-FCNM-Escuela de Matemática-Profesorado en Matemática para Tercer Ciclo de Educación


Básica y Educación Media pág. 89
Geometrı́a I. 8 ISOMETRÍAS

8.2. Rotaciones.
8.2.1. Rotación en el plano orientado.
Un ejemplo es útil ahora para entender la importancia de trabajar en el plano orientado. En este
caso el centro de rotación es O y el ángulo es π2 . En la figura adjunta, además de la ortogonalidad
indicada, suponga que, OM2 = OM1 = OM . La formulación: “Un punto M asociado a M 0 talque
OM 0 = OM y ∠M OM 0 = π2 ” es ambigua, dado que los puntos M1 y M2 cumplen con la condición.

M2 O M1

En cambio, en el plano orientado, la formulación: “Un punto M asociado a M 0 tal que OM 0 =


−−→ −−→
OM y (OM , OM 0 ) = π2 ” no deja ninguna duda: un solo punto cumple con la condición, M 0 = M1 .
Por lo que en adelante, se utilizará el plano orientado.

Definición 8.1 M 0 es la imagen de M (M 6= O) por la rotación de centro O y ángulo α si:


−−→ −−→
OM 0 = OM y (OM , OM 0 ) = α.
M0

α
O M

Notación: RαO es la rotación de centro O y ángulo α.

Puntos Invariantes:

Si α = 0 entonces todos los puntos son invariantes.

Si α 6= 0 entonces el centro O de la rotación es el único punto invariante.

Biyectividad y Rotación Inversa: RαO es biyectiva y la rotación inversa es la rotación de


centro O y de ángulo −α.

Prueba 4 Basta demostrar que: RαO (M ) = M 0 si y sólo si R−α 0


O (M ) = M .

UES-FCNM-Escuela de Matemática-Profesorado en Matemática para Tercer Ciclo de Educación


Básica y Educación Media pág. 90
Geometrı́a I. 8 ISOMETRÍAS

8.2.2. Propiedades Esenciales.


Teorema 17 La rotación conserva distancias y ángulos.

1. La rotación envı́a a un segmento AB a otro A0 B 0 , igual en longitud, siendo A0 imagen de A,


B 0 imagen de B. Mas aún el segmento AB se transforma en el segmento A0 B 0 .

2. La rotación transforma a una recta en otra, siendo el ángulo entre ellas α.

Prueba 5 Sea A0 = RO,α (A) y B 0 = RO,α (B), entonces, OA = OA0 , OB = OB 0 y ∠A0 OB 0 =


∠AOB 0 − ∠AOA0 = ∠AOB 0 − ∠BOB 0 = ∠AOB, ası́ por LAL el 4A0 OB 0 es congruente con
4AOB, por tanto, la longitud del segmento AB es la misma que la del segmento A0 B 0 .
A continuación demostramos que la imagen del segmento AB es el segmento A0 B 0 . Sea C
un punto del segmento AB o su prolongación, y C 0 = RO,α (C), se demuestra como antes que
A0 C 0 = AC y B 0 C 0 = BC, por consiguiente A0 C 0 + B 0 C 0 = AC + CB = AB = A0 B 0 , dado que
A, B y C estan alineados, por lo tanto, A0 C 0 + C 0 B 0 = A0 B 0 lo que demuestra que A0 , B 0 y C 0
alineados. En consecuencia, el segmento A0 B 0 es la imagen de AB y la recta A0 B 0 es la imagen de
la recta AB.
Ahora, determinamos el ángulo entre la recta y su imagen. Defina D como la intersección de
las rectas AB y A0 B 0 . Por la congruencia de los triángulos AOB y A0 OB 0 , se tiene que ∠BAO =
∠B 0 A0 O y por ende el cuadrilátero ADA0 O es cı́clico, por consiguiente, si E es un punto sobre la
recta AD entonces, ∠EDA0 = ∠AOA0 = α.

A0
C0 B
C
B0 A

Corolario 18 Suponga que: A0 = RO,α (A), B 0 = RO,α (B) C 0 = RO,α (C), demuestre que:
La imagen del punto medio de AB es el punto medio de A0 B 0 .
La rotación transforma a un cı́rculo de centro C y radio r en otro de igual radio y centro C 0 .
Si AB es diámetro del cı́rculo de centro C, entonces A0 B 0 es el diámetro del cı́rculo de centro C 0 .
La rotación transforma un triángulo ABC en otro igual A0 B 0 C 0
La rotación transforma a un ángulo en otro igual e igualmente orientado.
Dos rectas ortogonales tiene por imagen dos rectas ortogonales.
Dos rectas paralelas tienen por imagen dos rectas paralelas.

Prueba 6 Dejamos esta prueba como ejercicio.

UES-FCNM-Escuela de Matemática-Profesorado en Matemática para Tercer Ciclo de Educación


Básica y Educación Media pág. 91
Geometrı́a I. 8 ISOMETRÍAS

Ejemplo 8.4 Considere dos triángulos rectángulos isósceles ABC y AB 0 C 0 como en la figura
adjunta. Demuestre que BB 0 = CC 0 y que las rectas BB 0 y CC 0 son ortogonales.

Solución 1 Dado que por hipótesis AB = AC, AB 0 = AC 0 y el ∠BAC = ∠B 0 AC 0 = π2 , entonces,


C = RA, π2 (B) y C 0 = RA, π2 (B 0 ). Ası́ aplicando las propiedades de conservación de distancias y
ángulos de las isometı́as se concluye que BB 0 = CC 0 y como el ángulo de rotación es π2 , BB 0 ⊥ CC 0 .

B0

C0 B

Ejemplo 8.5 Utilización de una rotación Sea ABCD un cuadrado de centro O. M es un


punto del segmento AB, y N es un punto del segmento AD tal que DN = AM . Considere la
rotaci’on de centro O y de ángulo π2 en sentido positivo. Demuestre que OM = ON y el ∠M ON =
π
2
.

Solución 2 Recuerde que en todo cuadrado las diagonales son mediatrices mutuamente, ası́ que
OA = OD y el ∠AOD = π2 , en consecuencia D = RO, π2 (A), de la misma forma se verifica que:
A = RO, π2 (B), ası́ por las propiedades de conservación de la rotación la imagen del segmento AB se
transforma en el segmento DA, y por consiguiente M 0 = RO, π2 (M ) es el punto del segmento DA tal
que DM 0 = AM , por lo tanto M 0 = N , por definición de rotación se tiene que OM = OM 0 = ON
y el ∠M OM 0 = ∠M ON = π2 .

A M
B

N O

D C

Ejemplo 8.6 En la figura adjunta, ABCD es un rombo y los triángulos DCI y BCJ son equiláte-
ros. Defina una transformación geométrica y demuestre que A, I y J están alineados.

UES-FCNM-Escuela de Matemática-Profesorado en Matemática para Tercer Ciclo de Educación


Básica y Educación Media pág. 92
Geometrı́a I. 8 ISOMETRÍAS

Solución 3 La presencia de dos triángulos equiláteros del mismo vértice C nos sugiere definir
la rotación de centro C y ángulo π3 . Por esta rotación B es la imagen de J, D la imagen de I.
Ahora definimos A0 como la imagen de A, por consiguiente, el triángulo CAA0 es equilátero. Esto
demuestra que B, D y A0 están alineados, dado que pertenecen a la mediatriz de AC, puesto que:
BA = BC, DA = DC y A0 A = A0 C. Ahora solo falta considerar la rotación inversa de centro C
y ángulo − π3 para demostrar que J = RC,− π3 (B), I = RC,− π3 (D), A = RC,− π3 (B) están alineados
las propiedades de conservación de colinealidad de las isometrı́as.

A B
I

D
C

Ejemplo 8.7 Dada unaTcircunferencia Γ de centro C y radio CO. Demuestre que: Si Γ 0 =


RO,α (Γ ), P ∈ Γ , Q ∈ Γ Γ 0 y P 0 = RO,α (P ) entonces, P , Q y P 0 están alineados.

Solución 4 Por propiedades de conservación de isometrı́as sabemos que Γ 0 es la circunferencia


de centro C 0 = RO,α (C) y radio C 0 O, ya que O es el punto invariante de la rotación. Además,
dado que las rotaciones conservan ángulos se tiene que el ∠P CO = ∠P 0 C 0 O = 2θ, entonces,
el ∠OQP = 180◦ − θ por ser inscrito en la circunferencia Γ cuyo ángulo central es ∠P CO,
y el ∠P 0 QO = θ por ser inscrito en la circunferencia Γ 0 cuyo ángulo central es ∠P 0 CO. Por
consiguiente, ∠OQP + ∠P 0 QO = 180◦ − θ + θ = 180◦ y por lo tanto, P , Q y P 0 son colineales.


C C0 P0

θ
1800 − θ
Q

UES-FCNM-Escuela de Matemática-Profesorado en Matemática para Tercer Ciclo de Educación


Básica y Educación Media pág. 93
Geometrı́a I. 8 ISOMETRÍAS

8.3. Simetrı́as.
Estudiaremos en este apartado dos tipos de simetrı́as, las centrales y las axiales. En las primeras
la simetrı́a ocurre respecto a un punto denominado centro, mientras que en las segundas la simetrı́a
se da respecto a una recta que es denominada eje de simetrı́a.

8.3.1. Simetrı́as Centrales


Considere los puntos del plano P y un punto C que denominaremos centro. La simetrı́a central
de centro C es una transformación del plano que envı́a cada punto p del plano en otro punto p0
del plano de manera tal que el punto C es el punto medio del segmento [p, p0 ].

P C P0

Hay muchas figuras usuales en Geometrı́a que poseen un centro de simetrı́a en el sentido que
cuando se les aplica la transformación la figura queda globalmente invariante, es decir que a pesar
de que sus puntos no quedan fijos, la figura como tal si permanece la misma. Tal es el caso de las
siguientes figuras:

1. Los paralelogramos respecto a su centro O, esto es respecto al punto en donde se cortan sus
diagonales. Recuerde que los rombos, rectángulos, rombos y cuadrados son casos particulares
de paralelogramos.

Q0 P0

O
P Q

2. Las circunferencias cuando se simetrizan respecto a su centro.

8.3.2. Propiedades de las simetrı́as centrales.


Las simetrı́as centrales tienen la propiedad de conservar:

1. Distancias, es decir la distancia entre dos puntos y sus correspondientes imágenes es la misma,
es decir d(P, Q) = d(P 0 , Q0 ).

2. Puntos medios. Esta propiedad significa que dados dos puntos P y Q y sus respectivas imágenes
P 0 y Q0 , el punto medio M del segmento [P, Q] tiene por imagen a M 0 que es el punto medio de
[P 0 , Q0 ].

3. El alineamiento de puntos. Esto significa que si tres o más puntos están alineados, sus imágenes
están igualmente alineadas.

4. Los ángulos y su orientación. Es decir, si P[ 0 Q0 R0 = α


QR = α, entonces P\

UES-FCNM-Escuela de Matemática-Profesorado en Matemática para Tercer Ciclo de Educación


Básica y Educación Media pág. 94
Geometrı́a I. 8 ISOMETRÍAS

5. Las áreas. Es decir si una figura F tiene área A, la imagen F 0 que se obtiene después de la
simetrización es siempre igual a A.

Prueba 7

1. Tomando en cuenta que la distancia de d(C, P ) = d(C, P 0 ) y d(C, Q) = d(C, Q0 ) y la igualdad


de ángulos P[
CQ = P\0 CQ0 , por el criterio LAL de congruencia de triángulos se deduce que los

triángulos 4P CQ y 4P 0 CQ0 son congruentes y en consecuencia d(P, Q) = d(P 0 , Q0 ).

Q0 P0

P Q

2. Como en la prueba anterior, los triángulos 4P CM y 4M CQ son respectivamente congruentes


con los triángulos 4P 0 CM 0 y 4M 0 CQ0 , respectivamente; en consecuencia P 0 M 0 = P M =
M Q = M 0 Q0 , de donde se deduce que P 0 M 0 = M 0 Q0 , es decir M 0 es el punto medio de P 0 Q0 .

Q0 M0 P0

P M Q

3. Sean P , Q y R tres puntos alineados, condición que significa que P Q + QR = P R. Por la


primera de las propiedades de las simetrı́as centrales, ya demostrada, de conservar distancias,
sabemos que P 0 Q0 = P Q, Q0 R0 = QR y P 0 R0 = P R, en consecuencia se tiene: P 0 Q0 + Q0 R0 =
P Q + QR = P R = P 0 R0 y en consecuencia P 0 , Q0 yR0 están alineados

4. Considere el ángulo P[QR. Verifique que el triángulo P QR es congruente con el triángulo P 0 Q0 R0


y que en consecuencia sus ángulos correspondientes son iguales, en particular tendremos la
igualdad P[
QR = P\ 0 Q0 R0 .

Además, las simetrı́as centrales tiene la propiedad de que la composición con ella misma da
como resultado la identidad. Es decir si Sc es la simetrı́a de centro c, entonces Sc2 (P ) = I(P ) = P.;
por otra parte las simetrı́as centrales tiene la propiedad de que el segmento[P, Q] y el segmento
imagen [P 0 , Q0 ] son paralelos.

Se dice que un punto P es invariante por una transformación cuando su imagen P 0 coincide con
P . Se dice también que el punto es un punto fijo por la transformación. En el caso de las simetrı́as
centrales el único punto que permanece invariante por ella es el centro mismo de la transformación;
para cualquier otro punto P del plano se tiene que su imagen P 0 es diferente a P .

UES-FCNM-Escuela de Matemática-Profesorado en Matemática para Tercer Ciclo de Educación


Básica y Educación Media pág. 95
Geometrı́a I. 8 ISOMETRÍAS

8.3.3. Simetrı́as Ortogonales


Sea D una recta del plano. Llamamos simetrı́a axial ortogonal o reflexión a la transformación
del plano que a cada punto P del plano lo transforma en el punto P 0 que satisface las siguientes
condiciones:

1. El punto medio del segmento [P, P 0 ] pertenece a la recta D.

2. El segmento [P, P 0 ] es ortogonal o perpendicular a la recta D.

P0

8.3.4. Propiedades de las simetrı́as ortogonales


Las propiedades son similares a las propiedades de las simetrı́as centrales, salvo que la propie-
dad de conservación de los ángulos se reduce sólo a la conservación de la medida de los ángulos, no
ası́ la orientación de los mismos que en las simetrı́as ortogonales la orientación se invierte. Todas
las demás propiedades son idénticas.

Una figura se dice que tiene un eje de simetrı́a ortogonal de eje D, cuando la figura queda
invariante al aplicarle la simetrı́a. De las figuras usuales en geometrı́a las siguientes poseen eje de
simetrı́a:

1. Los rombos. Sus ejes de simetrı́a son sus diagonales.

P R D’

UES-FCNM-Escuela de Matemática-Profesorado en Matemática para Tercer Ciclo de Educación


Básica y Educación Media pág. 96
Geometrı́a I. 8 ISOMETRÍAS

SD (P ) = R, SD (Q) = Q, SD (R) = P y SD (S) = S.


SD0 (P ) = P , SD0 (Q) = S, SD0 (R) = R y SD0 (S) = Q.

2. El triángulo 4ABC isósceles en A tiene como eje de simetrı́a la altura que sale de A.

3. El triángulo equilátero tiene sus tres alturas como ejes de simetrı́a.

4. El rectángulo admite como ejes de simetrı́a a las rectas que unen los puntos medios de sus
lados paralelos.

5. El cuadrado admite como ejes a sus diagonales y a las rectas que unen puntos medios de sus
lados paralelos.
S R

E1
D1
E2

D2

P Q

En este caso, por ejemplo, se tienen las siguientes correspondencias:


SD1 (Q) = Q, SD1 (S) = S, SD1 (R) = P y SD1 (P ) = R.
SE1 (Q) = R, SE1 (S) = P , SE1 (R) = Q y SE1 (P ) = S.

Las simetrı́as ortogonales además tienen, como las simetrı́as centrales, la propiedad de que la
2
composición con ella misma es la identidad; es decir que SD (P ) = I(P ) = P ; sin embargo, en las si-
metrı́as ortogonales, un segmento [P, Q] y su segmento imagen [P 0 , Q0 ], en general, no son paralelos.

En las simetrı́as ortogonales los puntos fijos son todos los puntos que pertenecen a la recta de
simetrización.

8.4. La Isometrı́a Directa más simple.


Teorema 19 Dados dos segmentos congruentes AB y A0 B 0 , existe una isometrı́a directa que trans-
forma a A en A0 y a B en B 0 . Además, de todas las isometrı́as que cumplen esta condición, las
más simple siempre será una rotación, o una traslación en caso excepcional.
−−→
Prueba 8 Si ABB 0 A0 forma un paralelogramo, claramente la traslación de vector AA0 transforma
A en A0 y B en B 0 . Si en cambio ABA0 B 0 forma un paralelogramo, tomando O como la intersección
de AA0 con BB 0 , una rotación de centro O y ángulo 180◦ funciona.23
A0 B0 B’ A’
O

A B A B

23
Observe que esta rotacion es una reflexión puntual respecto a O.

UES-FCNM-Escuela de Matemática-Profesorado en Matemática para Tercer Ciclo de Educación


Básica y Educación Media pág. 97
Geometrı́a I. 8 ISOMETRÍAS

En cualquier otro caso (es decir, AB ∦ A0 B 0 ), sea P la intersección de AB con A0 B 0 , y sea O


la otra intersección de los circuncı́rculos de los triángulos 4AA0 P y 4BB 0 P . Note que OP AA0 y
OP BB 0 son cuadriláteros concı́clicos, entonces ∠OAB = ∠OA0 B 0 y ∠OBA = ∠OB 0 A0 , y dado
que por hipótesis AB = A0 B 0 se concluye que 4ABO ≡ 4A0 B 0 O y tienen igual orientación. Note
además que si llamamos α al ángulo que la recta AB forma al rotarse con centro P hasta coincidir
con la recta A0 B 0 , de nuevo por los cuadriláteros concı́clicos se obtiene α = ∠AOA0 = ∠BOB 0 .
Ası́, una rotación de centro O y ángulo α funciona como Isometrı́a Directa que transforma a A en
A0 y a B en B 0 .

B0
O
α
α A0

α
P A B

8.5. Descomposición de Traslaciones y Rotaciones como Composición


de Simetrı́as Axiales.
Ya se estudió el hecho que una Simetrı́as Axial es una Isometrı́a Indirecta, esto quiere decir que
a pesar que mantiene invariante las distancias cambia la orientación de los ángulos. En particular,
si los vértices de un triángulo 4ABC se recorren en sentido antihorario, los vértices del triángulo
imagen tras una simetrı́a axial 4A0 B 0 C 0 se recorren en sentido horario. Si ahora se aplica otra
simetrı́a axial (o la misma reflexión axial anterior, si se quiere) al 4A0 B 0 C 0 , el triángulo imagen
4A00 B 00 C 00 cumple que sus vértices se leen en sentido antihorario. Ası́, el 4ABC y el 4A00 B 00 C 00 son
triángulos congruentes y con la misma orientación, esto significa que existe una isometrı́a directa
que transforma al 4ABC en el 4A00 B 00 C 00 .

Por otra parte, ya se sabe que una isometrı́a directa siempre puede resumirse a una sola rota-
ción, o una sola traslación en caso excepcional. A pesar que no es evidente, las Reflexiones Axiales
pueden utilizarse para generar Rotaciones y Traslaciones, lo que las convierte en las isometrı́as
más fundamentales. Esto significa que cualquier isometrı́a puede escribirse como la composición
de reflexiones axiales. Más impresionante aún es que la cantidad de reflexiones axiales necesarias
es únicamente 2.

Teorema 20 Una Traslación puede descomponerse como el producto de dos Reflexiones Axiales.
−→
Prueba 9 Si la traslación está determinada por el vector AB, al tomar dos rectas paralelas entre
−→
sı́ l1 y l2 , y perpendiculares a AB, tales que la distancia desde l1 hasta l2 es igual a la mitad de
−→
la longitud del vector AB, se cumple que la composición de reflexiones axiales Rl2 ◦ Rl1 es igual a
la traslación T− →.
AB

UES-FCNM-Escuela de Matemática-Profesorado en Matemática para Tercer Ciclo de Educación


Básica y Educación Media pág. 98
Geometrı́a I. 8 ISOMETRÍAS

P P2 P1

A B
2d

l1 d l2

Teorema 21 Una Rotación puede descomponerse como el producto de dos Reflexiones Axiales.

Prueba 10 Dada la rotación de centro O y ángulo α, se construyen dos rectas l1 y l2 que pasan
por O, y que cumplen que el ángulo desde l1 hasta l2 es igual a α2 . Es relativamente sencillo
mostrar que la composición de reflexiones axiales Rl2 ◦ Rl1 es igual a la rotación de centro O y
ángulo α.
B2 l2

A2
A1
B1

l1
α O

B
A

A partir de estos dos teoremas anteriores es evidente el siguiente corolario:

Corolario 22 Toda Isometrı́a Directa puede descomponerse como el producto de 2 Reflexiones


Axiales.

Más aún, una Isometrı́a Indirecta o bien es una Reflexión Axial o bien la composición de una
Isometrı́a Directa con una Reflexión Axial, por lo tanto, toda Isometrı́a Indirecta puede escribirse
como la composición de a lo sumo tres Reflexiones Axiales. De aquı́ se deriva el siguiente resultado,
que es fundamental en la teorı́a de Transformaciones Geométricas.

Teorema 23 Toda Isometrı́a puede escribirse como la composición de a lo sumo 3 Reflexiones


Axiales.

8.6. Problemas de Traslaciones.


1. Sean ABCD un paralelogramo de centro O, la paralela a BD que pasa por A corta a la
paralela AC que pasa por D en el punto E. ¿Cuáles son las imágenes de A y D bajo la
−−→
traslación del vector EO?

2. Considere el triángulo 4ABC isósceles en A, I el punto medio de BC, D = t−→ (B) y


AC
E = t−→ (D). Estudie la naturaleza de los cuadriláteros ABDC y DECI.
BI

UES-FCNM-Escuela de Matemática-Profesorado en Matemática para Tercer Ciclo de Educación


Básica y Educación Media pág. 99
Geometrı́a I. 8 ISOMETRÍAS

3. Sean ABCD un paralelogramo de centro O, M y N los puntos medios de los segmentos DA


y BC respectivamente. ¿Son verdaderas o falsas las siguientes afirmaciones?

a) Los puntos M, O y N son alineados.


b) Los segmentos AB y CD son simétricos con respecto a la recta M N .
c) La traslación que transforma D en C, transforma M en N .
d ) La rotación de centro O y ángulo AOD,
\ transforma C en B.
−→ −−→
e) La traslación de vector AC es igual a la traslación de vector BD.

4. Construya un triángulo 4ABC, en seguida el punto D, simétrico de A con respecto a B, y


−→
el transformado E de B por la traslación de vector AC. Mostrar que el triángulo 4ABC es
el transformado del triángulo 4BDE por una traslación la cual debe precisarse el vector.

5. Sea 4ABC un triángulo rectángulo en B, y H la proyección ortogonal de B sobre AC. Se


denotan por A0 y C 0 los puntos tales que HBAA0 y HBCC 0 son dos paralelogramos. ¿Cuál
es la naturaleza del cuadrilátero CAA0 C 0 ?
−−→
Sugerencia: Utilice la traslación de vector BH.

6. Traslación de una circunferencia:

a) Sean Γ y Γ0 dos circunferencias de igual radio, centros respectivos O y O0 , y que se


cortan en A y B. Demuestre que Γ0 = T− −→ (Γ).
OO0
b) Demuestre que A0 = T−−→ (A) es el punto diametralmente opuesto a B en Γ0 .
OO0
c) Dado un punto M sobre Γ, sea M 0 = T−−→ (M ) perteneciente a Γ0 . Demuestre que
OO0
M A ⊥ BM 0 .
d ) Demuestre que A es el ortocentro del triángulo variable 4BM M 0 .
e) A partir de lo anterior, demuestre que si tres circunferencias de radios iguales pasan
por un punto H, y A, B, C, son las otras intersecciones al tomarlas dos a dos, H es el
ortocentro del 4ABC. Demuestre además que el circunradio del 4ABC tiene la misma
longitud que los radios de las tres circunferencias.

7. Dado un punto fijo O, una circunferencia de radio constante que pasa por O gira tomando
O como centro de rotación. Sea l una recta fija; para cada posición de la circunferencia, se
trazan las rectas tangentes a la circunferencia que son paralelas a l. ¿Qué lugar geométrico
describen los puntos de tangencia de estas rectas?

8. Dado un rectángulo ABCD, se construye en su exterior un triángulo cualquiera 4ABE.


Demuestre que la recta perpendicular a AE que pasa por C, la recta perpendicular a EB
que pasa por D, y la recta perpendicular a AB que pasa por E, concurren.
Pista: Hay una traslación que es la clave... y un ortocentro.

9. Dado un cuadrado ABCD, sean E y F puntos sobre los lados AB y BC, tales que AE = BF .
Sea H la intersección de AF con CE. Demuestre que H es el ortocentro del 4DEF .
Pista: Trabaje con una rotación de centro O para comparar los triángulos 4ABF y 4DAE,
donde O es el centro del cuadrado.

UES-FCNM-Escuela de Matemática-Profesorado en Matemática para Tercer Ciclo de Educación


Básica y Educación Media pág.
Geometrı́a I. 8 ISOMETRÍAS

10. El cuadrado ABCD es de centro O y de lado a. Determine los puntos M sobre el lado AB,
N sobre el lado CD, tales que M N k BC y que minimicen la poligonal OM N B, y exprese
este mı́nimo en función de a.
Pista: Sea O0 = T−−→ (O).
BC

11. Consdiere dos circunferencias de radios iguales Γ de centro O, y Γ0 de centro O0 , que se cortan
en A y en B. Considere dos rectas paralelas l y l0 :

l pasa por A, corta nuevamente a Γ en P , y a Γ0 en P 0 ;


l0 pasa por B, corta nuevamente a Γ en Q, y a Γ0 en Q0 .

Demuestre que P P 0 Q0 Q es un paralelogramo.

12. Demuestre que la composición de dos traslaciones es otra traslación.

8.7. Problemas de Simetrı́as Centrales o Reflexiones Puntuales.


1. ¿Cuáles son las figuras simétricas de la figura siguiente con respecto a las cuatro esquinas
del rectángulo?

2. Sean las rectas d y d0 secantes en O y el punto I no perteneciente a ninguna de las rectas


anteriores.

a) Construya el punto intersección A de la recta d0 con la imagen de la recta d por la


simetrı́a de centro I .
b) La recta AI corta a d en B . Precise la posición de los puntos I, A y B.
c) Aplicación: Construya un segmento de punto medio I, con uno de sus extremos en d y
el otro en d0 .

3. Dado un triángulo 4ABC, sea M el punto medio de BC. D y E son las proyecciones
ortogonales respectivas de B y C sobre la mediana AM . Utilizando propiedades de simetrı́as
puntuales aplicadas a la simetrı́a respecto a M , demuestre que BD = CE, ie, los vértices B
y C equidistan de la mediana AM .

4. Sea ABCD un paralelogramo de centro O. Especificar las imágenes de los puntos A, B, C


y D por S0 .

5. El triángulo 4ABC es rectángulo en A; denotamos por D = SA (B) y E = SA (C). ¿Cuál es


la naturaleza del cuadrilátero BCDE?

UES-FCNM-Escuela de Matemática-Profesorado en Matemática para Tercer Ciclo de Educación


Básica y Educación Media pág.
Geometrı́a I. 8 ISOMETRÍAS

6. Si ABCD y AECF son paralelogramos, ¿qué tipo de cuadrilátero es BEDF ?

7. Dado un paralelogramo ABCD de centro O, se trazan dos rectas paralelas l y l0 pasando


respectivamente por A y C. Si l corta a la recta BC en E, y l0 corta a la recta DA en F ,
determine la naturaleza del cuadrilátero BEDF .

8. Dado el paralelogramo ABCD de centro O, sean I y J puntos en el exterior del paralelogramo


tales que 4ADI y 4CBJ son triángulos rectángulos isósceles, con ángulo recto respectivo
en D y B. Demuestre que I, O y J están alineados.

9. Dado un triángulo 4ABC, sean B1 y C1 las proyecciones ortogonales respectivas de B y C


sobre la mediana AA0 . Demuestre que BB1 CC1 es un paralelogramo.

10. Dado el triángulo 4ABC, encuentre el lugar geométrico de las simetrı́as con respecto a C
de los centros de las circunferencias que pasan por A y por B.

11. Dos circunferencias de radios iguales Γ y Γ0 son tangentes externamente en el punto T . Sean
l y l0 dos rectas que pasan por T .

l corta nuevamente a Γ en M y a Γ0 en M 0 .
l0 corta nuevamente a Γ en N y a Γ0 en N 0 .

a) Demuestre que M N M 0 N 0 es un paralelogramo.


b) ¿Qué condiciones hay que imponer a las rectas l y l0 para obtener especı́ficamente: un
rombo, un recctángulo, un cuadrado.
c) Demuestre que la composición de dos reflexiones puntuales es una traslación. En parti-
cular, si los centros de reflexión coinciden, el resultado es la transformación identidad.

8.8. Problemas de Rotaciones.


1. Dada la figura siguiente, construya las imágenes del cuadrado, triángulo y segmento obtenidos
por la rotación de centro O y de ángulo 90◦ (en ambos sentidos).

2. Sean d una recta, O un punto no perteneciente a d y r la rotación de centro O y ángulo 120◦ ,


en el sentido directo. Construir, en el siguiente orden:

a) La proyección ortogonal de H de O sobre d.


b) La imagen H 0 de H por r.

UES-FCNM-Escuela de Matemática-Profesorado en Matemática para Tercer Ciclo de Educación


Básica y Educación Media pág.
Geometrı́a I. 8 ISOMETRÍAS

c) La imagen d0 de d por r .
d ) Si denotamos por I el punto intersección de d y d0 , mostrar que O, I, H y H 0 están
sobre un mismo cı́rculo.

3. Sean: ζ un cı́rculo de centro O, r la rotación de centro O y ángulo de 50◦ . El punto A


pertenece a ζ, denotemos por B la imagen de A por r y por C la imagen de B por r.

a) Ilustre la situación anterior.


b) Exprese en grados los ángulos del triángulo 4ABC.

4. Trace el segmento AB de 4cm. Ubique el punto O de tal forma que la rotación de centro O
y ángulo 90◦ , muevan el punto A al punto B. ¿Cuál es la distancia del punto O a la recta
AB.

5. a) ¿Cómo trazar los vértices de un hexágono regular, utilizando sólo el compás?


b) Aplicación: Dados dos puntos O y A: Construir utilizando sólo el compás, el punto
simétrico de A con respecto a O y el transformado A por la rotación de centro O y
ángulo 120◦ .

6. Sea el cuadrado ABCD de centro O en sentido directo. M un punto del segmento AD y N


su imagen por una rotación de 90◦ en sentido directo y de centro O.

a) Demostrar que N pertenece al segmento AB.


b) Comparar las distancias de BM y CN , los ángulos AOM
\ y BON
\ y las áreas de los
triángulos 4BDM y 4CAN .

7. Dada una recta l y un punto O que no pertenece a l, demostrar que las imágenes de l al
hacer todas las rotaciones de centro O, son las tangentes a una circunferencia fija.

8. Dada una recta l y un punto O fuera de ella, se construye un triángulo equilátero 4OAB de
tal forma que A varı́a sobre l. ¿Qué lugar geométrico describe B?

9. Sobre los lados de un n-ágono regular A1 A2 · · · An se dibujan puntos B1 , B2 , . . . , Bn tal que


A1 B1 = A2 B2 = · · · = An Bn . Demuestre que B1 B2 · · · Bn también es n-ágono regular.

10. Dado un cuadrado ABCD nombrado en sentido horario, sea I un punto en su interior tal
que 4DCI es equilátero, y J un punto en su exterior tal que 4BCJ también es equilátero.
Demuestre que A, I, J, están alineados. Para ello:
π
a) Construya las imágenes de J, I, A tras la rotación RC3 y demuestre que estas tres
imágenes están alineadas.
b) Concluya por propiedades de rotaciones que J, I, A, están alineados.

11. Los triángulos 4ABC y 4AB 0 C 0 son triángulos rectángulos isósceles y con la misma orien-
tación. Demuestre que 4ABB 0 ≡ 4ACC 0 .

UES-FCNM-Escuela de Matemática-Profesorado en Matemática para Tercer Ciclo de Educación


Básica y Educación Media pág.
Geometrı́a I. 8 ISOMETRÍAS

12. Rotación de una circunferencia tomando un punto sobre la circunferencia como


centro de rotación:
Sean Γ y Γ0 dos circunferencias de radios iguales y centros respectivos O y O0 . Suponga
que las circunferencias se cortan en los puntos A y B. Se traza una recta por B que corta
nuevamente a Γ en P , y a Γ0 en P 0 . Demuestre que

a) 4AOP ≡ 4AO0 P 0 .
b) 4AOO0 ' 4AP P 0 .
c) La rotación de centro A y ángulo ∠OAO0 transforma a O en O0 y a P en P 0 .
d ) Concluya a partir de lo anterior que esta rotacion transforma a Γ en Γ0 .

13. ABCD es un cuadrado de centro O. Sean I, J, K, L, los puntos medios de los lados AB,
BC, CD, DA. Demuestre que las rectas DI, AJ, BK, CL, se cortan en cuatro puntos que
son los vértices de un cuadrado. Para ello, utilice una rotación de centro O y ángulo 90◦ .

14. Sea ABCD un cuadrado en sentido directo. Sea M un punto de la recta BC. La recta
perpendicular en A a AM corta a CD en M 0 . Se designa por r a la rotación de centro A y
de ángulo 90◦ en sentido directo.

a) Precisar las imágenes de las rectas AM y BC por r.


b) ¿Cuál es la naturaleza del triángulo 4AM M 0 ?
C M B

D A

M0

15. Sean los triángulos rectángulos isósceles 4OAB y 4OA0 B 0 , con vértice común O. Sea I el
punto medio de A0 B. Queremos establecer que las rectas OI y AB 0 son ortogonales (dicho
de otra manera, que la mediana que sale de O en el triángulo 4OBA0 es la altura que desde
O se traza en el triángulo 4OB 0 A).

a) Sea r una rotación de 90◦ en sentido directo de centro O y C el punto r (B 0 ). Demuestre


que O es el punto medio de A0 C y que BC ⊥ AB 0 .
b) ¿Qué representa la recta OI en el triángulo 4BA0 C? Deducir que OI ⊥ AB 0 .

UES-FCNM-Escuela de Matemática-Profesorado en Matemática para Tercer Ciclo de Educación


Básica y Educación Media pág.
Geometrı́a I. 8 ISOMETRÍAS

A0 A
O

B0

16. El cuadrado ABCD de lado a tiene por centro O. Otro cuadrado se ubica de tal forma que
uno de sus vértices coincide con O. ¿Cuál es el valor del área común de los dos cuadrados?

17. El triángulo 4OAB es isósceles respecto a O, y ABCD es un paralelogramo. Se construye


E = R∠AOB
O (D). Demuestre que 4BCE es isósceles con respecto a B y que ∠AOB = ∠CBE.

18. El paralelogramo y los cuadrados:

a) Dado un paralelogramo ABCD, sea I un punto en el exterior tal que 4ABI es un


triángulo rectángulo isósceles. Se construye también en el exterior el cuadrado BCF E.
Demuestre que la rotación de centro I y ángulo de 90◦ transforma a D en E.
b) Dado el paralelogramo ABCD, se construyen exteriormente los cuadrados ABP Q,
BCSR, CDU T , DAW V , cuyos centros respectivos son I, J, K, L. Utilizando el pro-
blema anterior, demuestre que IJKL es un cuadrado.

19. El triángulo 4ABC es equilátero en sentido directo y M es un punto interior en el triángulo


tal que M A = 5, M B = 4, M C = 3. Determine la medida del lado del 4ABC, para ello:
π
a) Sea N = RA3 (M ), determine las longitudes de los lados del triángulo 4CM N y precise
la naturaleza de este triángulo.
b) Con la ayuda de las relaciones métricas en el triángulo 4ACN , deduzca la longitud del
lado del triángulo equilátero 4ABC.

20. La recta l y el punto A son fijos. Un punto M es escogido sobre l, se trazan dos circunferencia,
una de centro A y de radio AM , y la otra de centro M y radio AM . ¿Cuál es el lugar
geométrico de los puntos de intersección de las circunferencias cuando M varı́a sobre l?

21. Sea ABCD un cuadrado. Se construye un rectángulo AP QR tal que:

P y R están sobre los lados AB y AD, respectivamente;


AP = DR.

El problema tiene por objeto mostrar que los segmentos CQ y P R son perpendiculares e
iguales. Para ello:

1° Demostrar que B, Q, D, están alineados.


2° Construya O, el punto medio BQ.

UES-FCNM-Escuela de Matemática-Profesorado en Matemática para Tercer Ciclo de Educación


Básica y Educación Media pág.
Geometrı́a I. 8 ISOMETRÍAS


3° Determine las imágenes de C y Q tras la rotación R90
O .

4° Concluya.

22. Sea ABCD un cuadrado tal que un punto P en su interior cumple que P A = 1, P B = 2 y
P C = 3. Determine la medida del ángulo ∠AP B. Para ello

1° Defina P 0 = R90
B (P ).

2° Estudie la naturaleza del triángulo 4BP P 0 y determine sus ángulos.


3° Analice la naturaleza del triángulo 4P CP 0 .
4° Con la información anterior, deduzca la magnitud de ∠AP B.

23. El triángulo equilátero 4ABC tiene sentido horario y está inscrito en una circunferencia de
centro O. Considere un punto M en el arco AB que no contiene a C. Se trata de comparar
M A + M B y M C.24

a) Calcular la medida del ángulo AM


\ C.
b) Sea M 0 la imagen de M por la rotación r de centro A y de ángulo 60◦ (en sentido
horario). ¿Cuál es la naturaleza del triángulo 4AM M 0 ? Compare entonces AM con
M M 0.
c) Precisar r (B).
d ) Demostrar con ayuda de los literales anteriores que M 0 pertenece al segmento M C.
Concluya que M A + M B = M C.

24. Los puntos E y F están sobre los lados BC y CD de un cuadrado ABCD y son tales que
BE = CF . Demostrar que el punto de intersección H de las rectas BF y DE es el ortocentro
del triángulo 4AEF . Para ello: Sea O el centro de una cuadrado y r la rotación de centro
O y ángulo 90◦ en sentido directo. Buscar las imágenes de AE y DE por r y explotar que
luego de esta transformación, una recta y su imagen....

25. Puntos de Fermat: Dado un triángulo 4ABC, se busca el punto P en su interior tal que la
suma de segmentos P A + P B + P C sea mı́nima. Para ello:

1° Dado un punto P cualquiera en el interior del triángulo 4ABC (consideramos los vértices
◦ 60◦
se nombran en sentido antihorario), construya P 0 = R60 0
A (P ) y C = RA (C). ¿El triángulo
4ACC 0 tiene algo especial?
2° Compare la suma P A + P B + P C, la poligonal BP P 0 C 0 y BC 0 . ¿A qué recta deberı́a
pertenecer P para minimizar la suma de segmentos en cuestión?
3° Si B 0 es un punto al exterior del 4ABC y cumple que 4BAB 0 es triángulo equilátero,
¿qué relación guarda el punto P que minimiza la suma con la recta CB 0 .
4° Concluya la ubicación exacta del punto P buscado.
5° Demuestre que si sobre los lados de un triángulo, se construyen exteriormente triángulos
equiláteros, las rectas que van de un vértice del triángulo dado al vértice más alejado del
triángulo equilátero construido sobre el lado opuesto, son concurrentes. Este punto recibe
el nombre del Primer Punto de Fermat; el Segundo Punto de Fermat se obtiene cuando
24
Este problema también puede ser resuelto utilizando el Primer Teorema de Ptolomeo.

UES-FCNM-Escuela de Matemática-Profesorado en Matemática para Tercer Ciclo de Educación


Básica y Educación Media pág.
Geometrı́a I. 8 ISOMETRÍAS

se dibujan tres triángulos equiláteros pero hacia el “interior” del triángulo dado, y se
definen las rectas tal como en el caso anterior. En ambos casos, las rectas que concurren
se relacionan una respecto a la otra con un giro de 60◦ de centro el punto de concurrencia.
−→ −−→
26. Dados dos vectores no paralelos AB y A0 B 0 , demuestre que la rotación que transforma a uno
en el otro tiene por centro a la intersección de las mediatrices de los segmentos AA0 y BB 0 .

27. Demuestre el teorema conocido como la Recta de Steiner. Para ello

1° Dado un triángulo 4ABC cuyo ortocentro es H, demuestre que los circunradios de los
triángulos 4ABH, 4BCH y 4CAH, son iguales.
2° Dado un punto P sobre el circuncı́rculo del triángulo 4ABC, construya PA , PB , PC , las
reflexiones axiales del punto P con respecto a las rectas BC, CA, AB.
3° Utilizando la descomposición de una rotación como el producto de dos reflexiones axiales,
demuestre que PA se transforma en PB mediante una rotación de centro C y ángulo de
rotación 2∠BCA.
4° A partir de lo anterior y utilizando propiedades referentes a la rotación de una circunfe-
rencia cuando el centro de rotación pertenece a ella misma, demuestre que PA , H y PB
están alineados.
5° Repitiendo el proceso anterior, demuestre que PB , H y PC están alineados, y análogamente
PC , H y P A .
6° Concluya que PA , PB , PC y H están alineados. Esta recta se conoce como la Recta de
Steiner respectiva al punto P .

28. Demuestre que la composición de dos rotaciones es otra rotación cuyo ángulo de rotación es
igual a la suma docde los ángulos de rotación de cada rotación componente, a excepción que
esta suma sea 360◦ o un múltiplo de 360◦ en cuyo caso el resultado es una traslación.

a) Demuestre el caso cuando los ángulos de rotación suman 360◦ k con k ∈ Z.

En los siguientes dos literales, se asume lo contrario al literal anterior: que la suma de
los ángulos no es de la forma 360◦ k.

b) Si los dos centros de rotación originales coinciden, la rotación resultante también tiene
dicho punto como centro de rotación.
c) En caso contrario, si O1 y O2 son los centros de rotación originales, y los ángulos de
rotación respectivos son α y β, construya un cuadrilátero bisósceles OO1 O0 O2 de tal
forma que O1 O = O1 O0 , O2 O = O2 O0 , y además ∠OO1 O0 = α y ∠O0 O2 O = β. Observe
que al aplicar la primera rotación a O se transforma en O0 , y al aplicar la segunda
rotación a O0 se transforma en O, por lo que O queda invariante tras esa composición
de rotaciones y por tanto es el centro de rotación.

8.9. Problemas de Reflexiones Axiales.


1. Determine los ejes y centros de simetrı́a de cada figura:

UES-FCNM-Escuela de Matemática-Profesorado en Matemática para Tercer Ciclo de Educación


Básica y Educación Media pág.
Geometrı́a I. 8 ISOMETRÍAS

a) Un punto y un cı́rculo.
b) Un cı́rculo que pasa por dos puntos dados.
c) Un segmento y un punto.
d ) Dos rectas paralelas y un punto.

2. Dado un hexágono regular ABCDEF , encontrar seis rotaciones y seis simetrı́as que lo dejan
invariante.

3. ¿Puede suceder que al quitar 2011 puntos a un cı́rculo, se obtenga una figura que admita un
eje de simetrı́a?, ¿y un centro de simetrı́a?

4. Dadas dos circunferencias de radios iguales y centros O y O0 , demuestre que tanto la recta OO0
como la mediatriz del segmento OO0 funcionan como ejes de simetrı́a de la figura geométrica
formada por las circunferencias.

5. Sea ABCD un rombo: ¿Cuáles son los puntos simétricos de A, B, C y D:

a) con respecto a AC?


b) Con respecto a BD?

6. Construya con el compás el punto simétrico de un punto P con respecto a una recta dada,
con solo dos trazos.

7. Sea d una recta y Sd la reflexión de eje d. Para dos puntos A y B, se establece: A0 = Sd (A) y
B 0 = Sd (B). En cada uno de los casos siguientes, coloque los puntos A y B de manera que :

a) AA0 B 0 B sea un rectángulo;


b) AA0 BB 0 sea un rectángulo;
c) AB y A0 B 0 sean ortogonales.

8. El triángulo 4ABC es rectángulo en A. I y J son los puntos medios de BC y AC, finalmente


∆ k AC. P y Q son las intersecciones de ∆ con IC e IA, respectivamente.

a) Probar que AI y IC son simétricas con respecto a IJ.


b) Demuestre que el triángulo 4P IQ es isósceles.

9. Trace el cuadrado ABCD y determine en cada caso los ejes de simetrı́a que intercambia las
rectas D y D0 .

a) D = AC y D0 = BD.
b) D = AB y D0 = AD.
c) D y D0 son las medianas de un cuadrado.

10. Considere el triángulo 4ABC. Construya los puntos M que estén a igual distancia de AB y
AC, y además sean equidistantes de B y C.

11. Sean las rectas D y D0 tangentes al cı́rculo C de centro O. Encuentre los puntos M de la
recta ∆ , tales que la reflexión de eje OM intercambie D y D0 .

UES-FCNM-Escuela de Matemática-Profesorado en Matemática para Tercer Ciclo de Educación


Básica y Educación Media pág.
Geometrı́a I. 8 ISOMETRÍAS

12. En los siguientes problema conteste si es Verdadero o Falso:

a) Un triángulo equilátero admite un centro de simetrı́a.


b) Ningún triángulo tiene exactamente dos ejes de simetrı́a.
c) La figura siguiente, admite dos centros de simetrı́a.

d ) En una rotación de 60◦ , el centro de la rotación, un punto y su imagen tras la rotación,


son los vértices de un triángulo equilátero.
e) Si las tangentes en dos puntos A y B de un cı́rculo de centro O son perpendiculares,
entonces el triángulo 4AOB es rectángulo isósceles.
Para los ejercicios siguiente, considere el cuadrado ABCD de centro O, M y N los
puntos medios de los lado BC y DC respectivamente.
f ) La imagen de M por la reflexión de eje CA y la de C por la rotación de centro O y
ángulo de 45◦ coinciden.
g) La rotación de centro A y ángulo de 45◦ , transforma M en N .
h) OM y ON son las bisectrices de las diagonales del cuadrado.

13. A0 y C 0 son los simétricos de A y de C con respecto a la diagonal BD del paralelogramo


ABCD.

a) Sea O el centro de del paralelogramo. Demostrar que la simetrı́a de centro O transforma


A0 en C 0 . (Utilice este hecho para justificar que la imagen de la mediatriz de un segmento
es la mediatriz de su segmento imagen).
b) Deduzca que AA0 CC 0 es un rectángulo.

14. En la siguiente figura, ABCD es un rectángulo. Nos proponemos establecer que IJKL es un
rombo.

a) Sea ∆ la mediatriz de AB y S∆ la reflexión de eje ∆. Determine las imágenes de AK y


DI por S∆ y deducir que I y K pertenecen a ∆.
b) Demostrar que J y L pertencen a la mediatriz de BC.
c) Concluya.

UES-FCNM-Escuela de Matemática-Profesorado en Matemática para Tercer Ciclo de Educación


Básica y Educación Media pág.
Geometrı́a I. 8 ISOMETRÍAS

D C

K
L J
ED C
L
B
A
I

A B

15. Sea l una recta fija, y A y B dos puntos distintos que no pertenecen a l. Para cada punto
M en l, se construyen dos circunferencias: la primera centrada en A y que pasa por M , y
la segunda con centro en B pasando también por M . Estas dos circunferencias se cortan
nuevamente en M 0 . ¿Cuál es el lugar geométrico que describe M 0 cuando M varı́a?

16. Problemas de Optimización: En cada uno de los problemas siguientes, dé un algoritmo
para construir los puntos convenientemente.

a) Principio de Herón: Dada una recta l y dos puntos A y B que no pertenecen a ésta,
¿qué punto M en l minimiza la suma de segmentos AM + M B?
b) Dadas dos rectas l y l0 , sea M un punto variable sobre l, y M 0 un punto variable sobre l0 .
Fijado un punto A, ¿qué posiciones deben tomar M y M 0 para que la suma de segmentos
AM + M M 0 sea mı́nima?
c) Dadas dos rectas l y l0 , el punto M varı́a sobre l mientras que M 0 varı́a sobre l0 . Dados
dos puntos A y B, ¿qué posiciones deben ocupar M y M 0 para que la poligonal AM M 0 B
tenga longitud mı́nima? Considere dos casos: cuando l k l0 y cuando l ∦ l0 .
d ) Sean A y B dos ciudades divididas por un rı́o. Las alcaldı́as de las ciudades han de-
cidido construir un puente, pero si escogen una ubicación inadecuada, los costos se
elevarán innecesariamente. El costo depende principalmente de la longitud del recorri-
do. ¿Qué puntos deben escogerse como extremos del puente para minimizar el recorrido?
Nota: Suponga que el rı́o tiene orillas paralelas y el puente debe ser construido perpen-
dicular a las orillas.

UES-FCNM-Escuela de Matemática-Profesorado en Matemática para Tercer Ciclo de Educación


Básica y Educación Media pág.
Geometrı́a I. 8 ISOMETRÍAS

17. Sea ABCD un rectángulo, y se construyen las imágenes P y Q de B y D al hacer una


reflexión con respecto a AC. Precise la naturaleza de los cuadriláteros AP CQ y BP DQ.

18. Sean Γ y Γ0 dos circunferencias de igual radio y tangentes exteriormente en un punto I.


Sea Γ1 una circunferencia concéntrica a Γ y de radio mayor. Sea A uno de los puntos de
intersección de Γ0 con Γ1 , la recta AI corta nuevamente a Γ en B, y a Γ1 en C. Demuestre
que AI = IB = BC.
Pista: Utilice el eje de simetrı́a de las circunferencias para concluir un igualdad de segmentos,
y la simetrı́a puntual en I para la otra igualdad.

19. La carrera de las banderas: En el terreno rectangular ABCD con dimensiones AB = 90m y
BC = 60m, sea I el punto medio del lado AB. Cada concursante participa individualmente,
iniciando su recorrido en el punto I, debe colocar la primer bandera en la lı́nea AD, después
una segunda en la lı́nea DC, una tercera en la lı́nea CB y la última bandera en la esquina
A. ¿Dónde deben los concursantes colocar sus banderas para que el trayecto recorrido sea
mı́nimo? ¿cuál es la longitud de ese trayecto?

20. El Problema de Fagano: Dado un triángulo acutángulo 4ABC, sean P , Q, R, puntos sobre
los lados BC, CA, AB, respectivamente. Determine la ubicación exacta de los puntos P , Q,
R, para que el triángulo variable 4P QR tenga perı́metro mı́nimo. Para ello:

1° Construya P 0 , la reflexión de P con respecto a CA, y P 00 , la reflexión de P con respecto


a AB. Luego, compare la longitud de P 0 P 00 , la longitud de la poligonal P 0 QRP 00 y el
perı́metro del 4P QR.
2° Demuestre que independientemente de la ubicación de P , el ∠P 0 AP 00 es constante, ¿qué va-
lor toma? Determine a partir de esta información qué ubicación debe tener P para que
P 0 P 00 se minimice.
3° Fijado el punto P del numeral anterior, construya convenientemente los puntos Q y R
para que la poligonal P 0 QRP 00 se minimice.
4° Concluya. ¿Qué nombre recibe el triángulo encontrado?
5° ¿Qué sucede si el triángulo 4ABC no es acutángulo?

UES-FCNM-Escuela de Matemática-Profesorado en Matemática para Tercer Ciclo de Educación


Básica y Educación Media pág.
Geometrı́a I. 8 ISOMETRÍAS

21. Golf en miniatura. Se trabajará con la hipótesis siguiente: “Las trayectorias de la bolas
de golf tienen las mismas propiedades que las taryectorias de las bolas de billar”. Encontrar
para cada una de las bolas 1 y 2 una trayectoria que permita entrar en el agujero T .

BOLA DOS

BOLA UNO

UES-FCNM-Escuela de Matemática-Profesorado en Matemática para Tercer Ciclo de Educación


Básica y Educación Media pág.

También podría gustarte